Problemas Algebra
Problemas Algebra
Problem-solving
Todos estos documentos se actualizan constantemente. ¡No utilices una versión anticuada! Descarga totalmente gratis la última versión de los
documentos en los enlaces superiores.
[Link]
Índice.
1 Operaciones con números. →
1.1 Operaciones con números naturales.
1.2 Operaciones con números enteros.
1.3 Operaciones con fracciones.
1.4 Operaciones con decimales.
1.5 Operaciones con porcentajes.
1.6 Aproximación de números.
1.7 Proporcionalidad numérica.
2 Potencias y radicales. →
2.1 Potencias con exponente natural.
2.2 Potencias con exponente entero.
2.3 Raíces y exponentes fraccionarios.
3 Exponenciales y logaritmos. →
3.1 Problemas con exponenciales.
3.2 Problemas con logaritmos.
4 Polinomios. →
4.1 Operaciones con polinomios.
4.2 Factorización de polinomios.
4.3 Teorema del residuo.
4.4 Funciones polinómicas.
4.5 Las fórmulas de Vieta.
4.6 Las identidades de Newton.
5 Fracciones algebraicas. →
6 Expresiones algebraicas. →
7 Ecuaciones y sistemas de ecuaciones. →
7.1 Ecuaciones y sistemas en general.
7.2 Ecuaciones polinómicas.
7.3 Sistemas de ecuaciones polinómicas.
7.4 Ecuaciones irracionales.
7.5 Sistemas de ecuaciones irracionales.
CANGURO N5 2019 #1
1.1.2 MF
Un tren de juguete tarda exactamente 1 minuto y 11 segundos en completar una vuelta a
un circuito. ¿Cuánto tiempo invertirá en seis vueltas?
CANGURO N5 2019 #2
1.1.3 MF
Evalúa:
1997 1998 1998 1998 1998 1997 1997 1997
1.1.4 MF
¿Cuál es el resultado de 2014 2014 2014 2014 ?
1.1.5 MF
Calcula 2 2008 2008 8 .
(A) 4016 (B) 16064 (C) 20080 (D) 64256 (E) 80020
1.1.6 MF
Determina el número mayor:
AMC 8 2017 #1
1.1.7 MF
Calcula:
100 98 96 94 92 90 ... 8 6 4 2
1.1.9
Calcula 1999 999 99 9
(A) 1900 (B) 1090 (C) 1000 (D) 1990 (E) 1009
CANGUR N1 1999 #1
1.1.10
Calcula
2 0 0 1
CANGUR N1 2001 #1
1.1.11
Determina la operación que da un resultado más grande:
(A) 111 (B) 900 (C) 909 (D) 990 (E) 999
CANGUR N1 2004 #1
1.1.13
Calcula:
10 100 20 80
(A) 20000 80000 (B) 2000 8000 (C) 2000 80000 (D) 20000 8000
(E) 2000 800
CANGUR N1 2004 #3
1.1.14
20042005 dividido entre 2005 es igual a
(A) 11 (B) 101 (C) 1001 (D) 10001 (E) La división no es exacta
CANGUR N1 2005 #1
1.1.15
Determina el resultado de la operación 20 0 6 20 0 6
CANGUR N2 2006 #2
1.1.16
Para conseguir que 1+20♣9-2=2008 sea correcto, ¿Por qué elemento deberemos
sustituir ♣?
CANGUR N1 2008 #4
1.1.17
Si sabemos que ♣ + ♣ + 6 = ♣ + ♣ + ♣ + ♣ , ¿Cuál es el número que corresponde a ♣ ?
CANGUR N1 2010 #1
1.1.18
Determina el resultado de la operación 12 23 34 45 56 67 78 89 ?
(A) 404 (B) 304 (C) 405 (D) 396 (E) 389
CANGUR N2 2010 #1
1.1.19
1.1.20
Determina todos los valores posibles que se pueden obtener añadiendo paréntesis a la
siguiente expresión:
23 45
Determina
100 100 3 100 100 3
1.1.23
Calcula:
20 40 60 80 100 120 10 30 50 70 90 110
MOEMS E 2011 1A
1.1.24
Calcula:
87 76 65 54 43 32 21 10
MOEMS E 2012 1A
1.1.25
81 18 72 27 63 36 54 45 4
1.1.26
55 11 44 22 33 33 22 44 11 55
MOEMS E 2008 #1A
1.1.27
8 4 8 3 8 2 8 1
MOEMS E 2009 #1A
1.1.28
87 76 65 54 43 32 21 10
1.1.29
47 8 8 27 26 8
MOEMS E 2012 #2A
1.1.30
10 234 10 342 10 423
1.1.32
9 26 83 55 45 17 74 91
1.1.33
100 91 82 73 64 55 46 37 28 19 10
MOEMS E 2015 #3A
1.1.34
18 40 12 40 20 28 20 12
1.1.35 M
15 20 47 20 38 20 100 12 100 5 100 3
1.1.36
102 203 304 405 506 607 708 809 901
1.1.38
908 807 706 605 504 403 302 201
MOEMS E 2015 5A (Follow-Up)
1.1.39
531 642 753 864 975
MOEMS E 2018 3A
1.1.40
3 5 5 7 7 9 9 11
MOEMS E 2019 3A
1.1.41
30 8 40 11 22 19
MOEMS E 2019 4A
1.1.42
10 18 32 45 19 31 10 25 20
1.1.44
1974 6 1969 4 45 6 50 4
1.1.45
732 935 868 265
MOEMS E 2019 4A
1.1.46
47 12 12 53 53 8 8 47
MOEMS E 2020 5A
1.1.47
91 82 73 64 55 46 37 28
1.1.48
268 1375 6179 168 1275 6079
MOEMS SET 6 OLYMPIAD 1 1A
1.1.49
100 98 96 94 92 90 ... 8 6 4 2
MOEMS SET 7 OLYMPIAD 3 3C
1.1.50
185 278 579 85 178 279
MOEMS SET 10 OLYMPIAD 1 1A
1.1.51
1 3 5 7 9 11 13 15 17 19
1.1.52
Si sabemos que 1 2 3 .. 28 29 30 456 , determina el valor de
31 32 33 .. 58 59 60
1.1.53
17 13 6113 22 13
AMC 8 2000 #1
1.2 Operaciones con números enteros.
1.2.1 MF
Determina el número de valores enteros de x satisfaciendo x 3 .
1.2.2 MF
1 3 5 ... 2017 2019 2 4 6 ... 2016 2018
(A) -1010 (B) -1009 (C) 1008 (D) 1009 (E) 1010
AMC 8 2018 #5
1.2.3 MF
Determina
4 1 2 3 4 5 6 7 ... 1000
AMC 8 2013 #3
1.2.4 MF
1990 1980 1970 1960 ... 20 10
(A) -990 (B) -10 (C) 990 (D) 1000 (E) 1990
1.2.5 MF
Determina el valor mínimo que podemos obtener multiplicando tres números diferentes
del conjunto
8 , 6 , 4 , 0 , 3 , 5 , 7
(A) -336 (B) -280 (C) -210 (D) -192 (E) 0
AMC 8 2000 #7
1.2.6 MF
1 2 3 4 ... 98 99
AHSME 1999 #1
1.2.7 MF
En el esquema siguiente, las sumas de las filas, de las columnas y de las diagonales son
el mismo número. Determina el producto a b c
1.3.1
1 1 1 1
2 1 3 1 4 1 ... 10 1
2 3 4 10
1.3.2
Calcula el siguiente producto:
1 1 1 1 1 1
1 1 1 1 1 1
1 2 3 4 5 6
AMC 8 2018 #2
1.3.3
1 3 2 4 3 5 97 99 98 100
...
2 2 3 3 4 4 98 98 99 99
1.3.4
Determina:
1 2 3 4 5 6 7 8
1 2 3 4 5 6 7 8
(A) 1020 (B) 1120 (C) 1220 (D) 2240 (E) 3360
AMC 8 2017 #5
1.3.5
3 5 7 9 11
9 11 3 5 7
AJHSME 1985 #1
1.3.6
1 1 1 1
1 1 1 ... 1
2 3 4 10
1 1 1 10 11
(A) (B) (C) (D) (E)
10 9 2 11 2
AJHSME 1985 #9
1.3.7
2
2
1
3
AJHSME 1986 #6
1.3.8
¿Qué fracción del siguiente rectángulo de 12 por 18 está sombreada?
(A) 1/108 (B) 1/18 (C) 1/12 (D) 2/9 (E) 1/3
1.3.9
1 2 3
10 20 30
(A) 0.1 (B) 0.123 (C) 0.2 (D) 0.3 (E) 0.6
AJHSME 1988 #3
1.3.10
164 es…
1.3.12
2 4 6
10 100 1000
(A) 0.012 (B) 0.0246 (C) 0.12 (D) 0.246 (E) 246
AJHSME 1989 #2
1.3.13
2 3 4 1 1 1
2 3 4
AJHSME 1989 #8
1.3.14
1
1
3
1
1
2
(A) 1/3 (B) 2/3 (C) 3/4 (D) 3/2 (E) 4/3
1.3.15
¿Qué fracción del cuadrado está sombreada?
(A) 1/3 (B) 2/5 (C) 5/12 (D) 3/7 (E) 1/2
AJHSME 1990 #3
1.3.16
Tenemos un triángulo pintado de gris, y a cada paso pintamos de blanco el triángulo
central de cada triángulo interior, tal y como se muestra en la figura. Después de cinco
pasos, ¿Qué fracción del triángulo original se mantendrá pintada de gris?
(A) 1/1024 (B) 15/64 (C) 243/1024 (D) 1/4 (E) 81/256
1.3.17
1 2 3 4 5 6 7 8 9 55
10 10 10 10 10 10 10 10 10 10
9
(A) (B) 6.4 (C) 9 (D) 10 (E) 11
2
AJHSME 1994 #2
1.3.18
1 9 9 7
10 100 1000 10000
(A) 0.0026 (B) 0.0197 (C) 0.1997 (D) 0.26 (E) 1.997
AJHSME 1997 #1
1.3.19
¿Qué fracción del cuadrado está sombreada?
(A) 5/12 (B) 1/2 (C) 7/12 (D) 2/3 (E) 5/6
AJHSME 1998 #3
1.3.21
1 1 1 1
2 1 3 1 4 1 ... 10 1
2 3 4 10
1.3.22
Determina la fracción asociada a la parte sombreada en el siguiente cuadrado:
(A) 1/6 (B) 1/7 (C) 1/8 (D) 1/12 (E) 1/16
1.3.23
Determina la fracción sombreada si continuamos la pauta que se muestra en el siguiente
esquema hasta el octavo paso:
(A) 3/8 (B) 5/27 (C) 7/16 (D) 9/16 (E) 11/45
AMC 8 2006 #9
1.3.25
Determina la fracción asociada a la zona gris respecto del total de la superficie.
AMC 8 2008 #6
1.3.26
El punto O es el centro de un octágono regular ABCDEFGH, y X es el punto medio del
lado AB . Determina la fracción asociada a la zona sombreada:
(A) 11/32 (B) 3/8 (C) 13/32 (D) 7/16 (E) 15/32
AMC 8 2015 #2
1.3.27
Determina la fracción de una cuadrícula 6 5 en la que trazamos un triángulo con
vértices A=(1,3), B=(5,1) y C=(4,4), tal y como se muestra en la imagen:
1 1 1 1
(A) (B) (C) 0 (D) (E)
64 16 16 64
ASHME 1979 #6
1.3.29
2003 2003 2003 2003 2003
La fracción es igual a
2003 2003
1.3.30
1111 3333 6666
Es sabido que 11 . ¿Cuánto vale ?
101 101 303
1.3.31
2004 2 2 2004
El número es igual a
1002
(A) 8 (B) 2006 (C) 4012 (D) 8024 (E) mayor que 10000
1.4.2 MF
Determina el producto 8 0.25 2 0.125
1 1 1
(A) (B) (C) (D) 1 (E) 2
8 4 2
AJHSME 1988 #2
1.4.3 MF
100 19,98 1,998 1000
(A) 1,9982 (B) 19,982 (C) 199,82 (D) 19982 (E) 199802
AJHSME 1998 #7
1.5 Operaciones con porcentajes.
1.5.1 MF
Carlos toma el 70% de un pastel entero. María toma una tercera parte del resto. ¿Qué
porción del pastel queda sin tomar?
(A) 10% (B) 15% (C) 20% (D) 30% (E) 35%
1.5.2 MF
Supongamos que a es el 150 % de b . ¿Qué porcentaje de a es 3b ?
2
(A) 50 (B) 66 (C) 150 (D) 200 (E) 450
3
AMC 12A 2019 #2
1.5.3 MF
Una gran urna contiene 100 bolas, de las cuales un 36% son rojas y el resto son azules.
¿Cuántas bolas azules hay que quitar para que el porcentaje de bolas rojas en la urna sea
del 72%? (No se quita ninguna bola roja)
1.5.4 MF
El diagrama de sectores muestra qué transporte utilizan los estudiantes de mi instituto.
Aproximadamente el número de los que vienen en bici dobla a los que usan el
transporte público y casi el mismo número viene en coche que a pie. El resto usa
patinete. ¿Qué porcentaje usa patinete?
CANGURO N6 2020 #4
1.6 Aproximación de números.
1.6.1 MF
¿Cuantos números enteros hay en el intervalo 20 21 , 20 21 ?
(A) 9 (B) 10 (C) 11 (D) 12 (E) 13
1.6.2 MF
5
¿Cuántos números hay en el intervalo entre y 2 ?
3
AMC 8 2000 #3
1.6.3 MF
1 1 1
¿ El número pertenece al intervalo , ?
5 7 2
1.6.4 M
Determina el número de cubos perfectos entre 28 1 y 218 1 , inclusive.
17.2 MF
Los equipos A y B juegan en una liga de baloncesto. Cada partido se gana o se pierde,
no hay empates. El equipo A ha ganado los 2 / 3 de los partidos que ha disputado, y el
equipo B ha ganado las 5 / 8 partes de los partidos que ha disputado. Además, el equipo
B ha ganado 7 partidos más y ha perdido 7 partidos más que A. ¿Cuántos partidos ha
disputado el equipo A?
1.7.3 MF
El área de una pizza de radio 4 pulgadas es un N por ciento mayor que el área de una
pizza de radio 3 pulgadas. ¿Cuál es el entero más próximo a N?
1.7.4 MF
5
Alicia dispone de dos recipientes. El primero está lleno de agua hasta sus partes y el
6
segundo está vacío. Vierte todo el agua del primer recipiente en el segundo, y observa
3
que el segundo recipiente queda lleno hasta sus partes. Determina la razón entre el
4
volumen del primer recipiente y del segundo.
5 4 7 9 11
(A) (B) (C) (D) (E)
8 5 8 10 12
AMC 12B 2019 #1
1.7.5 MF
Sam condujo 96 millas en 90 minutos. Su velocidad media durante los primeros 30
minutos fue 60 mph (millas por hora), y su velocidad media durante los segundos 30
minutos fue de 65 mph. ¿Cuál fue su velocidad media, en mph, durante los últimos 30
minutos?
1.7.6 MF
La razón de w a x es 4:3, la razón de y a z es 3:2, y la razón de z a x es 1:6. ¿Cuál es la
razón de w a y?
(A) 4:3 (B) 3:2 (C) 8:3 (D) 4:1 (E) 16:3
1.7.7 MF
Sean los puntos A, B y C alineados en este orden a lo largo de un camino recto donde la
distancia de A a C es de 1800 metros. Ina corre el doble de rápido que Eve, y Paul corre
el doble de rápido que Ina. Los tres corredores empiezan a correr al mismo tiempo, Ina
empezando en A y corriendo hacia C, Paul empezando en B y corriendo hacia C, y Eve
empezando en C y corriendo hacia A. Cuando Paul se encuentra con Eve, da media
vuelta y corre hacia A. Sabiendo que Paul e Ina llegan a B al mismo tiempo, determina
la longitud, en metros, de A a B.
AIME II 2018 #1
1.7.8 MF
René marcó con la mayor precisión posible dos puntos a y b en la recta numérica. ¿Cuál
de los puntos p, q, r, s, t en esta recta representa mejor su producto a·b ?
CANGURO N6 2020 #3
1.7.9 MF
Si D dromedarios pesan K kilos y E elefantes pesan lo mismo que M dromedarios,
¿cuántos kilos pesa un elefante, suponiendo que todos los animales de la misma especie
pesan lo mismo?
DK K E K M DM
(A) D K E M (B) (C) (D) (E)
EM DM DE K E
CANGURO N6 2020 #8
1.7.10
Las medidas de los tres ángulos de un triángulo mantienen la proporción [Link].
Determina la medida del ángulo mayor de dicho triángulo.
AMC 8 2017 #6
1.7.11
Dos bolsas contienen la misma cantidad de canicas, y cada canica puede ser azul o
verde. En la bolsa 1, la proporción de azules a verdes es 9:1, y en la bolsa 2, la
proporción de azules a verdes es 8:1. Si hay 95 canicas verdes en total, ¿Cuántas canicas
azules hay más en la bolsa 1 que en la bolsa 2?
1.7.12
Tenemos un mapa en el que una longitud de 12 centímetros representa 72 kilómetros.
¿Cuántos kilómetros representará una longitud de 17 centímetros?
AMC 8 2004 #1
1.7.13
En el mercado de frutas de Frank, 3 plátanos cuestan tanto como 2 manzanas, y 6
manzanas cuestan tanto como 4 naranjas. ¿Cuántas naranjas cuestan tanto como 18
plátanos?
(A) 6 (B) 8 (C) 9 (D) 12 (E) 18
2.1.2 MF
¿Qué número no es un cubo ni un cuadrado?
CANGUR N3 2015 #5
2.1.3 MF
¿Cuál es la cifra de las unidades del número 20152 20150 20151 20155 ?
CANGUR N3 2015 #1
2.1.4 MF
¿Cuál es la última cifra de 42015?
CANGUR N2 2015 #1
2.1.5 MF
999 999 999 999 999 999
¿Cuál de los siguientes números es igual a 1?
999 999 999
1010
(A) 99 (B) 99 1 (C) 910 (D) 109 (E)
9
CANGUR N1 2015 #15
2.1.6 MF
22014 22013
Simplifica el valor de la expresión 2013 2012 .
2 2
2.1.8 MF
Supongamos que n cumple 9n 9n 9n 32011. ¿Cuál es el valor de n ?
2.1.9 MF
Calcula el valor de la expresión
(A) 22048 (B) 24096 (C) 32048 (D) 34096 (E) 32048 22048
2.1.10 MF
Supongamos que los números racionales x , y i z cumplen x 2 yz 3 73 i xy 2 79 .
¿Cuál es el valor de xyz ?
2.1.11 MF
102000 102002
Determina
102001 102001
2.1.12 MF
22001 32003
Determina
62002
2.1.13 MF
¿Qué número no es un cuadrado perfecto?
(A) 12016 (B) 2 2017 (C) 32018 (D) 4 2019 (E) 52020
AMC 8 2016 #7
2.1.14 M
Ordena, de menor a mayor, los números 108 , 512 , 2 24 .
2.1.15 MF
2n4 2 2n
Simplifica
2 2 n 3
1 7 7
(A) 2 n 1 (B) 2 n 1 (C) 1 2 n (D) (E)
8 8 4
AHSME 1951 #24
2.1.16 MF
100 2 7 2 (70 11)(70 11)
Determina el valor de la siguiente expresión
70 2 112 (100 7)(100 7)
2.1.17 MF
Determina (1)1 (1) 2 (1)3 (1) 4 ... (1) 2005 (1) 2006
2.1.18 MF
10102 20202 30302
¿Cuál es el valor de la fracción ?
2020
(A) 2020 (B) 3030 (C) 4040 (D) 6060 (E) 7070
CANGURO N6 2020 #6, CANGUR 2n batx. 2020 #6
2.1.19 F
¿Qué número es mayor, 342 o 428 ?
2.1.20 MF
¿Qué número es más grande, 2255 o 5522 ?
2.1.21 MF
20212 1
Simplifica
2020
2.1.22 MF
33 33 33
AJHSME 1993 #7
2.1.23 MF
15
2
12
5
AJHSME 1989 #1
2.1.24 MF
66 66 66 66 66 66
2.1.25 MF
1530
4515
15 2
1 1
(A) (B) (C) 1 (D) 315 (E) 515
3 3
AHSME 1993 #3
2.1.26 MF
Determina el número de dígitos del producto 45 510
2.1.27 MF
Determina el número más grande:
CANGUR N4 2013 #1
2.1.28 MF
Determina el número de cifras del resultado de la multiplicación
2 5
22 5 55 2
CANGUR N4 2014 #7
2.1.29 MF
Simplifica la fracción
3 3
2008 2 2006 2
3 3
2007 2 2005 2
2.1.30 MF
Determina el orden correcto de los tres números 10 8 , 512 y 2 24
(A) 2 24 108 512 (B) 2 24 512 108 (C) 512 2 24 108 (D) 108 512 2 24
(E) 108 2 24 512
2.1.31 MF
2n 4 2 2n
Simplifica:
2 2n 3
1 7 7
(A) 2 n1 (B) 2n1 (C) 1 2n (D) (C)
8 8 4
ASHME 1951 #24
2.1.32 MF
10002
252 2 2482
AHSME 1984
2.1.33 MF
1 1 1 1
1 2 1 2 ...1 2 1 2
2 3 9 10
(A) 5/12 (B) 1/2 (C) 11/20 (D) 2/3 (E) 7/10
ASHME 1986 #9
2.1.34 MF
Si 21998 21997 21996 21995 k 21995, determina el valor de k.
2.1.36 MF
Determina el valor de n para el que se satisface 23 4n 85
2.1.37 F
100 81 64 49 36 25 16 9 4 1
2.1.38
10 7
5 10 4
2.1.39
4 4 9 4 49 99
(A) 1313 (B) 1336 (C) 3613 (D) 3636 (E) 129626
AHSME 45 #1
2.1.40
490 489 489 489 489 ?
2.1.41
2017 2 1
Calcula
2016 2018
2017 2016
(A) 2017 (B) 1 (C) (D) (E) Un altre valor
2018 2018
CANGUR 1r batx. 2017 #4
2.1.42
Calcula 13132 12122
MATHCOUNTS 2013
2.1.43
444 2 1112
Calcula
444 111
MATHCOUNTS 2013
2.1.44
Calcula 5627 5627 5637 5617
2.1.45
2427 827
?
911 911 911
2.1.46 M
a) Simplifica:
1
1 1
1 1
999 10002
2
2.1.47 M
Resuelve la siguiente ecuación:
2 5 3
5 3 x
2.1.48
720 2 180 2
540
2.1.49
2000 20002000 ?
(A) 20002001 (B) 40002000 (C) 20004000 (D) 40000002000 (E) 20004000000
AMC 10 2000 #2
2.1.50 MF
Determina el valor de x para el que se cumple 10 x 1002 x 10005 .
2.2.2 MF
Si x 0 , ¿Qué expresión será positiva?
x
(A) (B) x 2 (C) 2 x (D) x 1 (E) 3
x
x
2.2.3 MF
Después de simplificar, la expresión x 1 y 1 1
es igual a:
xy 1 x y
(A) x y (B) (C) xy (D) (E)
x y xy xy
AHSME 1952 #7
2.2.4 MF
a 4 b 4
La fracción es igual a:
a 2 b 2
2.2.5 MF
Calcula el valor de 2 32 3
1 1
AHSME 1958 #1
2.2.6 MF
Determina el valor de 20 1 52 0 1
5
1 5
(A) -125 (B) -120 (C) (D) (E) 25
5 24
AMC 12A 2015 #1
2.2.7 MF
Determina el valor de 2 2
2
1 7 9
(A) -2 (B) (C) (D) (E) 6
16 4 4
AMC 12B 2015 #1
2.2.8 MF
1
Si a b , determina el valor de a 3b ?
2
1 1
(A) (B) 8 (C) -8 (D) 6 (E)
8 6
CANGUR N4 2014 #3
2.2.9
8 5
45
2
2.3 Raíces y exponentes fraccionarios.
2.3.1 MF
La raíz cúbica de 33 es igual a:
3
CANGUR N4 2013 #4
2.3.2 MF
2.3.3 MF
4 4
Simplifica 3 6
a 9 6 3
a 9 . El resultado es:
AHSME 1956 #9
2.3.4 MF
1 1
La expresión 2 2 es igual a:
2 2 2 2
2
(A) 2 (B) 2 2 (C) 2 2 (D) 2 2 (E)
2
AHSME 1958 #5
2.3.5 F
Dados los números reales a, b, c 0 , resolver la ecuación
a bx b cx c ax b ax c bx a cx
Romania 1974
2.3.6 F
Calcula las soluciones reales de la ecuación 3
1729 x 3 x 19
2.3.7 F
Calcula las soluciones reales de la ecuación 4
97 x 4 x 5
2.3.9 MF
Determina el valor simplificado de la siguiente expresión:
1 1 3 1 3 5 1 3 5 7
2.3.10 M
Existen enteros a, b y c, todos ellos mayores que 1, tales que
a
N b N c N 36 N 25
2.3.11 F
3 2 2 3 2 2 es igual a
2.3.12 F
Supongamos que para cierto número real x se satisface la ecuación
49 x2 25 x2 3
Determina el valor de 49 x 2 25 x 2 .
Observación: Por estar este problema en el contexto de una prueba AIME, se sabe que el resultado será un entero entre 0 y 999.
AIME II 2005 #7
2.3.14 MF
Simplifica
34 37
7
2.3.15 M
Simplifica
53 10 6
2.3.16 F
810 410
?
84 411
?
5
2.3.18
Calcula 16 8 4
AMC 8 2017 #3
2.3.19
2 / 3
1
Calcula
125
AHSME 1961 #1
2.3.20
Demuestre que
1 1 1
... 9
1 2 2 3 99 100
2.3.21
Calcule
2.3.22 MF
Resuelve la ecuación
50 32 128 x
2
2.3.23 MF
4 3
¿Qué número es menor, 5 o 4 ?
2.3.24 MF
Si 8 x 32 , entonces x es igual a:
AHSME 1962 #4
2.3.25 MF
Determina el número de valores reales no negativos de x para los cuales 144 3 x en
un entero.
2.3.26 MF
Resuelve la siguiente ecuación:
2 548/ x
25 26/ x
5 2517 / x
2.3.27
502 142
2.3.28
217 219
10
2.3.29
75 48
3
2.3.30
72 32
128 8
2.3.31
98 32
2
2.3.32
52 2 482
2 5 2
2.3.33
Resuelve mentalmente:
3
25 2 3 1 x 3
2.3.34
1 1 1 1
1 3 3 5 5 7 7 9
2.3.35
95 38
?
9 5 34
2.3.36
Determina el valor de n para el que se cumple n 1222 1212 3
2.3.37
Simplifica
7 5 7 5
44
3 Exponenciales y logaritmos.
3.1 Problemas con exponenciales.
3.1.1 F
Resuelve la ecuación
4x 6x 9x
3.1.2 MF
Resuelve la ecuación 9 x 2 240 9 x
3.1.3 M
Determina todos los números reales x para los cuales
8 x 27 x 7
12 x 18 x 6
3.1.4 D
Determina todos los números reales x para los cuales
10x 11x 12x 13x 14x
3.1.5 F
Resuelve la siguiente ecuación:
2 x 3x 6 x 9 x
3.1.6 F
Si a, b, c son números reales tales que
2a 3b 7c 42
Determina
1 1 1
a b c
3.1.7 MF
Resuelve la ecuación 2 x5 2 x 33
3.1.8 MF
Resuelve la ecuación 95 x1 274
3.1.9 MF
Resuelve la ecuación 84 88 512 x
3.1.10 F
Si 2 x 4 y1 y 27 y 3x1 , ¿Cuánto vale x y ?
3.1.11
Si 21994 4997 8665 16 x , entonces
(A) x=997 (B) x=779 (C) x=499 (D) x=449 (E) x=399
3.1.12 F
Resuelve la ecuación 82 x4 44 x3
3.1.13 F
Si 21994 + 4997 + 8665=16x , entonces
(A) x=997 (B) x=779 (C) x=499 (D) x=449 (E) x=399
3.1.14 F
¿Cuántas soluciones tiene la ecuación 22 x 4 x1 ?
3.1.15 F
Supongamos que n cumple que 9n 9n 9n 32011. ¿Cual es el valor de n?
(A) 1005 (B) 1006 (C) 2010 (D) 2011 (E) Ninguna de las respuestas anteriores es
válida.
3.1.16 F
2x7 2 2x2
Simplifica:
5 2x 3
3.1.17 MF
Determina el valor de x para el que se cumple 3x 3x 3x 81
3.1.18 MF
Sean a y b números reales satisfaciendo las ecuaciones 3a 81b2 y 125b 5a3 .
Determina ab .
AMC 10 2007 #9
3.2 Problemas con logaritmos.
3.2.1
Demuestra que
log a b log b c log c d log d a 1
3.2.2 MF
Los valores de a en la ecuación log10 a 2 15a 2 son:
15 233 15 305
(A) (B) 10 , 5 (C) (D) 20
2 2
(E) Ninguno de los anteriores.
3.2.3 F
Tomamos dos números diferentes a y b del conjunto 2 , 22 , 23 , ... , 225 . ¿Cuál es la
probabilidad de que log a b sea un entero?
AIME II 2020 #3
3.2.5 F
Sean a y b dos números reales positivos tales que a b . Se sabe que a b 2a b
¿Cuál es el valor, redondeando a las décimas, de ln a 2 b 2 2 ln a b ?
3.2.6 F
Existe un único entero positivo n tal que
(A) 1 (B) log 5 6 (C) 2 (D) log 2 3 log 3 2 (E) log 2 6 log3 6
3.2.8 M
Los vértices de un cuadrilátero pertenecen a la gráfica de y ln x , y las abscisas de
91
estos vértices son enteros positivos consecutivos. El área del cuadrilátero es ln .
90
Determina la abscisa del punto situado más a la izquierda.
3.2.9 M
Supongamos que estamos ante una civilización marciana en la que todo logaritmo cuya
base no está especificada se supone de base b , para cierto b 2 fijo. Un estudiante
marciano escribe
3 log x log x 56
log log x ( x) 54
y encuentra que este sistema de ecuaciones tiene como única solución cierto número
real x 1 . Determina b .
AIME II 2019 #6
3.2.10 F
Sean x 1 , y 1 números reales positivos satisfaciendo log 2 x log y 16 , xy 64 .
2
x
Determina log 2
y
25 45
(A) (B) 20 (C) (D) 25 (E) 32
2 2
AMC 12A 2019 #12
3.2.11 M
¿Para cuantos valores enteros de x se puede construir un triángulo cuyos lados tengan
longitudes log 2 x , log 4 x , 3 ?
a b a log7 ( b )
, ab a log7 ( b )
para todos los reales a, b para los cuales tengan sentido estas expresiones. Definimos la
sucesión an de forma recursiva por a3 32 y
an n(n 1)an 1
3.2.13 F
Determina el valor de log 3 7 log 5 9 log 7 11 log 9 13 ... log 21 25 log 23 27
3.2.14 F
La solución de la ecuación log3 x 4 log 2 x 8 , donde x es un número real positivo que
no es 1/3 ni 1/2, se puede escribir como p / q , donde p y q son enteros coprimos
positivos. Determina p q .
3.2.15 M
Para cada par ordenado de números reales ( x, y) satisfaciendo
log 2 2 x y log 4 x 2 xy 7 y 2
Existe un número real K tal que
log 3 3x y log 9 3x 2 4 xy Ky 2
3.2.16 M
Determina:
log 2 80 log 2 160
log 40 2 log 20 2
3.2.18 MF
La solución de la ecuación 7 x7 8 x se puede expresar de la forma x log b 77 .
Determina b.
7 7 8 15 15
(A) (B) (C) (D) (E)
15 8 7 8 7
AMC 12A 2010 #11
3.2.19 MF
Si log x y 3 1 y log x 2 y 1 , determina logx y
1 1 3
(A) (B) 0 (C) (D) (E) 1
2 2 5
AMC 12A 2003 #17
3.2.20 F
¿Cuántas soluciones distintas tiene la ecuación x 2 4 x 5
x 2 x 30
1 ?
3.2.21 F
Si 2x =15, y 15y = 32, ¿cuál es el valor de xy?
CANGURO N6 2011 #3
3.2.22 F
Resuelve la ecuación 4 x 6 x 9 x
3.2.23 D
Sea x un número real diferente de 1/20 y 1/2 de forma que se satisface
m
El valor de log 20x 22 x se puede escribir como log10 , donde m y n son enteros
n
positivos coprimos. Determina m n .
AIME II 2022 #4
4 Polinomios y expresiones algebraicas.
4.1 Operaciones con polinomios.
4.1.1 MF
Si f ( x) a x 4 b x 2 x 5 y f (3) 2 , entonces f (3)
4.1.2 F
Sea f ( x) a x7 b x3 c x 5 , donde a, b, c son constantes. Si f (7) 7 , determina
f (7 ) .
4.1.3 MF
Si 3x 1 a7 x 7 a6 x6 ... a0 , determina a7 a6 ... a0
7
4.1.4 MF
Si x, y, z son números reales tales que
x 32 y 42 z 52 0
entonces x y z
AHSME 1997 #3
4.1.5 MF
Si k y h son constantes y x 2 k x 7 es equivalente a x 1x h , determina el valor
de k.
SAT
4.1.6 MF
Si x 2 3x 1 0 , entonces x 3 8x es igual a
4.1.7 M
Para cierto entero a, las ecuaciones 1988x 2 a x 8891 0 y 8891x 2 a x 1988 0
comparten una raíz común. Determina los posibles valores de a.
Canada 1988
4.1.8 MF
x 2 y 2 42
2 x 2 y ?
2
x y 3
4.1.9 F
Sean Px y Qx dos polinomios cuadráticos con coeficientes principales 2 y -2,
respectivamente. Las gráficas de ambos polinomios pasan por los dos puntos 16 , 54 y
20 , 53 . Determina Px Qx .
AIME I 2022 #1
4.2 Factorización de polinomios.
4.2.1 F
Uno de los factores de x 4 4 es:
4.2.2 F
Determina el número de enteros n entre 1 y 100 para los que x 2 x n factoriza en el
producto de dos factores lineales con coeficientes enteros.
AHSME 1989 #8
4.2.3 MF
Determina el valor de
3x 24x 1 3x 24x 1
para x 4
4.2.4
Sea f (x) un polinomio cúbico con raíces r1 , r2 , r3 tal que
1 1
f f
2 2 997
f (0)
1 1 1
Encuentre .
r1 r2 r2 r3 r3 r1
4.2.5 MF
Sea c la mayor solución de la ecuación x 2 20 x 13 0 . Determina el área de la
circunferencia de centro (c, c) y que pasa por el punto (13,7)
4.2.6 MF
Se muestra parte de la gráfica de f ( x) a x3 b x 2 c x d . ¿Cuál es el valor de b ?
4.2.7 F
Determina la suma de todos los posibles valores de k para los cuales los polinomios
x 2 3x 2 y x 2 5x k tienen una raíz en común.
4.2.8 M
Se toma aleatoriamente y de forma uniforme un número real a del intervalo 20,18 .
La probabilidad de que todas las raíces del polinomio
4.2.9 MF
El producto de tres enteros consecutivos es 8 veces su suma. ¿Cuál es la suma de sus
cuadrados?
AMC12B 2002 #7
4.2.10 M
Resuelve la ecuación
x 4 2 x3 6 x 2 2 x 1 0
4.2.11 M
Sean Q(z) y R(z) los únicos polinomios tales que
z 2021 1 z 2 z 1 Q( z) R( z)
4.2.12 M
Factoriza el polinomio x8 x 4 1 como producto de polinomios de menor grado.
4.3 Teorema del residuo.
4.3.1 MF
Determina el residuo al dividir x51 51 entre x 1 .
AHSME 1974 #4
4.3.2 MF
El residuo al dividir x13 1 entre x 1 es:
4.3.3 MF
Si 3x 3 9 x 2 k x 12 es divisible por x 3 , entonces también es divisible por:
4.3.4 F
El residuo R obtenido al dividir x100 entre x 2 3x 2 es un polinomio de grado menor
que 2. Este residuo R se puede escribir como:
(A) 2100 1 (B) 2100( x 1) ( x 2) (C) 2200( x 3) (D) x 2100 1 2 299 1
(E) 2100x 1 x 2
4.3.5 F
Sea P(x) un polinomio tal que cuando es dividido entre x 19 , el residuo es 99, y
cuando es dividido entre x 99 , el residuo es 19. Determina el residuo al dividirlo entre
x 19x 99 .
(A) x 80 (B) x 80 (C) x 118 (D) x 118 (E) 0
4.3.6 MF
Determina el valor de a para que el polinomio P( x) x1000 a x 2 9 sea divisible por
x 1.
4.3.7 F
Determina los valores de a y b si sabemos que el polinomio x2 x 6 es un factor de
2 x4 x3 ax2 bx a b 1.
4.4 Funciones polinómicas.
4.4.1 D
Determina el polinomio que tiene la raíz real mayor:
4.4.2 F
La figura muestra un arco de la parábola de ecuación y a x 2 bx c . ¿Cuál de los
siguientes números es positivo?
4.4.3 MF
Si definimos la función f de la forma f ( x) cx 6x 1 , con c 5 . Determina el
valor de la lista que es igual a f (7) .
Las fórmulas de Vieta son una herramienta muy potente para resolver problemas
de Teoría de Números. Por ejemplo, en los problemas 5.3.12, 6.37 o 6.75 del
libro de Aritmética.
x 2 ax b ( x p)( x q)
x 2 ax b ( x p)( x q) x( x q) p( x q) x 2 qx px p q
x 2 ( p q) x p q
Puesto que dos polinomios son iguales si y solo si tienen los mismos coeficientes, se
deducen directamente las fórmulas de Vieta para ecuaciones de segundo grado:
a ( p q )
b p q
Ejemplo 1.
Supongamos que p y q son las raíces de t 2 7t 5 0 . Encontrar p 2 q 2 .
Solución.
Aplicando las fórmulas de Vieta, tenemos que:
7 ( p q) 7 p q
5 p q 5 p q
49 7 2 ( p q) 2 p 2 q 2 2 pq p 2 q 2 2 5 p 2 q 2 10
39 49 10 p 2 q 2
Ejemplo 2.
Supongamos que m y n son las raíces de la ecuación 2 x 2 15x 16 0 .
1 1
Determinar .
m n
Solución.
En primer lugar, reescribimos la ecuación como un polinomio mónico:
2 x 2 15 x 16 0 2 15 16 15
2 x 2 15 x 16 0 x2 x 0 x2 x 8 0
2 2 2 2 2 2
Ejemplo.
Solución.
Aplicando las fórmulas de Vieta para polinomios de tercer grado:
2 ( p q r ) 2 p q r
3 p q p r q r 3 p q p r q r
4 p q r 4 p q r
( p 1)(q 1)(r 1) ( pq p q 1)(r 1) pqr pr qr r pq p q 1
pqr pr qr pq r p q 1 4 3 2 1 10
Fórmulas de Vieta generales.
p( x) an x n an 1 x n 1 ... a1 x a0
Observación.
De las fórmulas anteriores, la más útil es la última:
a0 (1)n an r1 r2 ...rn
Nos dice que el producto de todas las raíces y an es igual al término independiente, con
el signo correspondiente. Si los coeficientes del polinomio son enteros, entonces esta
última relación demuestra que cualquier raíz entera del polinomio tiene que ser
forzosamente uno de los divisores de an , con signo + o -.
4.5.1 F
Supongamos que las raíces de x3 3x2 4 x 11 0 son a, b y c, y que las raíces de
x3 r x 2 s x t 0 son a b , b c y c a . Encontrar t .
AIME 1996 #5
4.5.2 D
Para ciertos enteros a, b, c, d , definimos f ( x) x 2 ax b y g ( x) x 2 cx d .
Determina el número de ternas ordenadas a, b, c de enteros, con valores absolutos no
mayores de 10, para los cuales existe un entero d tal que
g f 2 g f 4 0
4.5.3 M
Sea P(x) un polinomio cuadrático con coeficientes complejos cuyo coeficiente de x 2
es 1. Supongamos que la ecuación PP( x) 0 tiene cuatro soluciones distintas:
x 3, 4, a, b . Determina la suma de todos los posibles valores de (a b)2 .
4.5.5 D
Supongamos que existen z1 , z2 , ..., z673 números complejos diferentes de forma que el
polinomio
x z1 3 x z2 3...x z6733
z z j k
1 j k 673
4.5.6 F
Determina el número de polinomios cuadráticos con coeficientes reales tales que su
conjunto de raíces es igual a su conjunto de coeficientes. Es decir: si el polinomio es
ax 2 bx c , a 0 , y las raíces son r y s , entonces la condición es a, b, c r , s.
4.5.7 M
Determine todas las raíces, reales o complejas, del siguiente sistema:
x y z 3,
x 2 y 2 z 2 3,
x 3 y 3 z 3 3.
USAMO 1973 #4
4.5.8 MF
Dada la ecuación 2 x 2 13x 15 0 , si sabemos que una de las soluciones es 5,
determina la otra.
4.5.9 MF
Dada la ecuación 2 x 2 3x 20 0 , si sabemos que una de las soluciones es -4,
determina la otra.
4.5.10 MF
1 1
Si a y b son las soluciones de la ecuación 3x 2 4 x 1 0 , determina .
a b
4.5.11 MF
1 1
Si a y b son las soluciones de la ecuación 2 x 2 11x 5 0 , determina .
a b
4.5.12 F
Si a y b son las soluciones de la ecuación 2 x 2 5x 3 0 , determina a 2 b 2 .
4.5.13 F
Sean a y b las soluciones de la ecuación 2 x 2 3x 5 0 . Determina el valor de
(a 1)(b 1) .
4.5.14 F
Las raíces de la ecuación x 2 x 2018 0 son m y n. ¿Cuál es el valor de n 2 m ?
(A) 2016 (B) 2017 (C) 2018 (D) 2019 (E) 2020
4.5.15 F
¿Cuál es el mayor valor del parámetro k tal que la suma de los cuadrados de las raíces
de la ecuación x 2 k x k 0 es igual a 3?
4.5.16 F
Dos números positivos, p y q, verifican la ecuación p 2 2 p q 2 2q 15 2 pq .
Hallar el valor de p q :
4.5.19 MF
Supongamos que m y n son las raíces de la ecuación 2 x 2 15x 16 0 . Determina el
1 1
valor de
m n
4.5.20 MF
Supongamos que m y n son las soluciones de la ecuación x 2 15x 28 0 . Determina
(m 1)(n 1) .
4.5.21 F
2003 1
Determina la suma de los recíprocos de las raíces de la ecuación x 1 0 .
2004 x
AMC 10A 2003
4.5.22 M
Determina todos los valores de m para los cuales las raíces de 2 x 2 m x 8 0 difieren
en m 1 .
HMMT 2001
4.5.23 F
1 1
Sean a y b las raíces de la ecuación x 2 m x 2 0 . Supongamos que a y b
b a
son las raíces de la ecuación x 2 p x q 0 . Determina q.
4.5.24 M
Todas las raíces del polinomio z 6 10 z 5 A z 4 B z 3 C z 2 D z 16 son enteros
positivos, posiblemente repetidos. Determina el valor de B.
(A) -88 (B) -80 (C) -64 (D) -41 (E) -40
4.5.25 F
Sean a y b las soluciones de la ecuación x 2 23x 3 0 . Determina a3 b3 .
4.6 Las identidades de Newton.
Las identidades de Newton permiten calcular la suma de potencias k-ésimas de las
raíces de un polinomio sin tener que determinarlas previamente.
Ejemplo resuelto.
Dado el polinomio p( x) x 2 2 x 6 , con raíces complejas a y b, determina a 4 b 4
Solución.
Aplicando las fórmulas de Vieta tenemos A a b 2 , y B ab 6 , luego:
P0 a 0 b 0 2
P1 a1 b1 A 2
P2 a 2 b 2 AP1 2 B 2 2 2 6 2
P3 a 3 b 3 AP2 BP1 2 (2) 6 2 4 12 16
P4 a 4 b 4 AP3 BP2 2 (16) 6 (2) 20
Solución.
Aplicando las fórmulas de Vieta deducimos que
A a b c 3 , B ab bc ac 6 , C abc 9
Y por tanto:
P0 a 0 b 0 c 0 3
P1 a1 b1 c1 A 3
P2 a 2 b 2 c 2 AP1 2 B 3 3 2 6 3
P3 AP2 BP1 CP0 3(3) 6 3 9 3 0
P4 AP3 BP2 CP1 3 0 6 (3) 9 3 45
P5 AP4 BP3 CP2 3 45 6 0 9 (3) 108
Ejemplo resuelto.
Dado el polinomio p( x) x 2 3x 2 4 x 8 , con raíces complejas a, b, c, determina
a 2 b2 c 2 y a 4 b4 c4
Solución.
Aplicando las fórmulas de Vieta deducimos que
A a b c 3 , B ab bc ac 4 , C abc 8
Y por tanto:
P0 a 0 b 0 c 0 3
P1 a1 b1 c1 A 3
P2 a 2 b 2 c 2 AP1 2 B (3) (3) 2 4 1
P3 AP2 BP1 CP0 (3) 1 4 (3) 8 3 33
P4 AP3 BP2 CP1 (3) 33 4 1 8 (3) 127
4.6.1 F
Suponiendo que
a b c 1
a 2 b2 c2 2
a 3 b3 c 3 3
Determina el valor de a b c .
4.6.2 M
Resuelve el sistema
x y z 1
2
x y z 35
2 2
x 3 y 3 z 3 97
4.6.3 D
Denotamos por sk la suma de las potencias k-ésimas de las raíces del polinomio
x3 5x2 8x 13 . En particular, s0 3 , s1 5 , y s2 9 . Sean a, b, c números reales
tales que sk 1 a sk bsk 1 c sk 2 para todo k 2,3,... Determina a b c .
Entonces:
an P1 an1 0
an P2 an1 P1 2an2 0
an P3 an1 P2 an2 P1 3an3 0
...
an Pn an1 Pn1 an2 Pn2 ... a1 P1 n a0 0
4.6.4 M
Considera los polinomios P( x) x 6 x5 x3 x 2 x y Q( x) x 4 x3 x 2 1 .
Si z1 , z2 , z3 , z4 son las soluciones de Q( x) 0 , determina Pz1 Pz2 Pz3 Pz4 .
AIME II 2003 #9
5 Fracciones algebraicas.
5.1 M
Dados a, b, c, d números reales diferentes de cero, definimos la función f como
ax b
f ( x)
cx d
5.2 F
Sea f ( x) x 2 1 x . Determina el valor de la suma
2
1 2 3 4 2017 2018
f f f f ... f f
2019 2019 2019 2019 2019 2019
1 20182 20202
(A) 0 (B) (C) (D) (E) 1
20194 20194 20194
AMC 12B 2019 #8
5.3 F
3x 6 12 x 3 y
Simplifica
x8 16 x 2 y 2
6 Expresiones algebraicas.
6.1 MF
Supongamos que a, b, c son números reales tales que
1 1 1 1 1 1
a b c 1 1 1
b c a a b c
Si abc 13 , determina a b c .
6.2 M
Sean a, b, c tres números reales diferentes de cero y distintos, tales que
1 1 1
a b c
b c a
6.3 F
Sea n un entero positivo que satisface la ecuación n 1!n 2! 440 n!
¿Cuál es la suma de los dígitos de n ?
6.4 M
Determina las tuplas a, b, c, d de números reales que satisfacen el siguiente sistema de
ecuaciones:
ab c d 3
bc d a 5
cd a b 2
da b c 6
IMO Team Selection Test 1, June 2013 #1 (Dutch team for IMO 2013)
6.5 MF
Si a, b, c, d son enteros que satisfacen a b 2 c d ¿cuál de los siguientes números no
puede ser el valor del producto a b c d ?
A) 54 B) 36 C) 34 D) 18 E) el valor es indeterminado
6.7 MF
2
Sea m 0 un entero, y sea n m . ¿Cuáles de las siguientes afirmaciones es
m
siempre cierta?
(A) Solo I (B) Solo II (C) Solo III (D) Solo II y III
6.8 F
Si x y z , x y r , entonces x y
z r2 z2 r z2 r2
(A) (B) (C) (D)
r2 4 z2 4
SAT
6.9 F
Si a b c , b c d , c d a , y b es un entero positivo, determina el mayor valor
posible de a b c d .
ARML 90 Individual #3
7 Ecuaciones y sistemas de ecuaciones.
COMC
7.1.2 M
Tenemos 15 números colocados en una rueda. Solo uno de los números es visible (un 10
en la parte superior). La suma de los números en cualesquiera 7 posiciones consecutivas
en la rueda (como las grises) es la misma en todos los casos. Si sumamos los 15
números de la rueda, ¿cuántos de los resultados 75, 216, 365 o 2020 son posibles?
A) ninguno B) 1 C) 2 D) 3 E) 4
7.1.3 MF
Supongamos que
1 144
2
1 53
1
2
2
3 x
Determina el valor de x.
(A) 3/4 (B) 7/8 (C) 14/15 (D) 37/38 (E) 52/53
7.1.4 F
1 / 2 1 / 2
Dada la matriz A , determina a, b tales que A3 A .
a b
Fuente: "Enjoy Solving Mathematics" en Facebook
7.1.5 D
Hallar todas las ternas de números reales a, b, c que cumplan el sistema
a b c 3
a
2 2 2 7
b c
2 a 2 b 3 / 4
OMEFL 2022 #3
7.2.2 MF
Resuelve la ecuación r 2
5r r 2 5r 3 4
7.2.3 M
Resuelve la ecuación xx 1x 2x 3 1 2712
7.2.4 F
Resuelve la ecuación x 2
x 1 x 2 x 2 12
7.2.5 F
Si f ( x) x 2 7 x k , y f (k ) 9 , entonces f (1) es
7.2.6 F
Determina todos los valores de r para los cuales r 2 5r r 2 5r 3 4
7.3 Sistemas de ecuaciones polinómicas.
7.3.1 F
3x 2 2 xy 336
Resuelve el sistema 2
2 y 3xy 84
7.3.2 F
r 2 2rs 16
Resuelve el sistema
rs 2s 2 4
7.3.3 M
x 2 y 2 x 2 y 2 2 xy 40
Resuelve el sistema:
xy x y 8
7.3.4 F
x 2 y 2 25
Determina las soluciones x, y del siguiente sistema
xy 12
Fuente: "Maths Solutions" en Facebook
7.4 Ecuaciones irracionales.
7.4.1 MF
10
Resuelve la ecuación 4x 3 7
4x 3
7.4.2 M
Resuelve la ecuación x 2 9 x 30 2 x 2 9 x 45
7.4.3 F
Resuelve mentalmente la siguiente ecuación: 3
25 2 3 1 x 3
7.4.4 D
Determina el producto de las raíces reales de la ecuación
x 2 18x 30 2 x 2 18x 45
AIME 1983 #3
7.5 Sistemas de ecuaciones irracionales.
7.5.1 MF
x y 7
Resuelve el sistema
3 x 4 y 14
7.5.2 MF
x y x y 30
Resuelve el sistema
x y x y 12
8 Funciones lineales, rectas, ecuaciones de primer grado.
8.1 F
Si m y b son números reales cumpliendo m b 0 , entonces la recta cuya ecuación es
y m x b no puede contener el punto
(A) (0,1997) (B) (0,-1997) (C) (19,97) (D) (19,-97) (E) (1997,0)
8.2 F
Sea f una función lineal con las siguientes propiedades:
f (1) f (2) , f (3) f (4) , f (5) 5
(A) f (0) 0 (B) f (0) 0 (C) f (1) f (0) f (1) (D) f (0) 5 (E) f (0) 5
8.3 MF
Sea f una función lineal para la que f (6) f (2) 12 . Determina f (12) f (2) .
8.4 MF
Si f ( x) a x b y f 1 ( x) b x a , con a y b reales, determina a b .
8.5 MF
Hace dos años, la edad de Gene era nueve veces la edad de Carol (la de aquel
momento). Actualmente, la edad de Gene es siete veces la edad de Carol. ¿Cuántos años
tienen que pasar para que la edad de Gene sea cinco veces la edad de Carol?
Mandelbrot
8.6 F
6751x 3249 y 26751
Resuelve el siguiente sistema
3249 x 6751y 23249
8.7 F
Los vértices de un triángulo rectángulo de lados 3-4-5 son los centros de tres
circunferencias externamente tangentes entre sí, como se muestra en la figura.
Determina la suma de las áreas de estos triángulos.
25 27
(A) 12 (B) (C) 13 (D) (E) 14
2 2
AMC 12A 2006 #13
8.8 MF
Se jugó un partido de rugby entre dos equipos: Los Cougars y los Panthers. Se anotaron
un total de 34 puntos, y los Cougars ganaron con una diferencia de 14 puntos. ¿Cuántos
puntos anotaron los Panthers?
8.9 F
Un grupo de chicos y chicas se dedican a lavar coches para ganar dinero para el viaje de
fin de curso. Inicialmente, el 40% del grupo son chicas, pero más tarde abandonan el
grupo dos chicas y se incorporan dos chicos, y entonces el 30% del grupo son chicas.
¿Cuántas chicas había inicialmente en el grupo?
8.10 F
Las ecuaciones 2 x 7 3 y b x 10 2 tienen la misma solución para x. Determina
el valor de b.
AHSME 1996 #2
8.12 F
En “Alicia en el país de las maravillas” de Lewis Carrol:
¨Tweedledum dice a Tweedledee: tu peso y el doble mio suman 361 libras, y
Tweleledee le responde: por el contrario, tu peso y el doble del mio suman 362 libras¨
Tweedledum (to Tweedledee): The sum of your weight and twice mine is 361 pounds.
Tweedledee (to Tweedledum): Contrawise, the sum of your weight and twice mine is
362 pounds.
8.13 F
En una competición matemática de 10 problemas, se obtiene 5 puntos por cada
respuesta correcta y se pierde 2 puntos por cada respuesta incorrecta. Si Olivia
respondió a todos los problemas y su puntuación fue de 29 puntos, ¿Cuántos problemas
resolvió correctamente?
8.14 MF
Si 3a 7b 1977 y 5a b 2007 , determina a b .
iTest 2007 #2
8.15 MF
Tenemos unas canicas rojas y azules en una bolsa. Si sacamos una canica roja, la
séptima parte de las que quedan en la bolsa son rojas. Si sacamos dos canicas azules (y
no sacamos la canica roja), la quinta parte de las que quedan son rojas. ¿Cuántas canicas
hay inicialmente en la bolsa?
AIME II 2021 #7
9 Inecuaciones.
9.1 F
Sea a IN con a 1 y la inecuación
x 2 a 1x 5a 2 a 2 1
Si el complemento del conjunto solución de la inecuación contiene exactamente 7
números enteros, calcule la suma de estos números.
9.2 MF
Alice, Bob y Charlie van de excursión y se preguntan como de lejos está el pueblo más
cercano. Alice dice: “Estamos al menos a 6 millas de distancia”, y Bob replica:
“Estamos como mucho a 5 millas de distancia”, y entonces Charlie remarca:
“Realmente el pueblo más cercano está a 4 millas como mucho”. Resulta que ninguna
de las tres afirmaciones es cierta. Sea d la distancia en millas al pueblo más cercano.
¿Cuál de los siguientes intervalos es el conjunto de todos los posibles valores de d ?
(A) 0,4 (B) 4,5 (C) 4,6 (D) 5,6 (E) 5,
9.3 M
Resuelve la siguiente inecuación: x 2
x
x 1 1
9.4 F
Un número positivo x satisface la inecuación x 2 x si y solo si:
1 1
(A) x (B) x 2 (C) x 4 (D) x (E) x 4
4 4
AHSME 1980 #6
9.5 F
El menor número real x que verifica la desigualdad x 2 2004 0 es:
CANGURO N6 2004 #3
10 Valor absoluto.
Definición de valor absoluto.
Propiedades básicas.
Ejemplo.
Resuelve la ecuación
x 2x 1 3
Solución.
x 2 x 1 3 (a)
x 2x 1 3
x 2 x 1 3 (b)
(a)
x 2x 1 3 x 3 2x 1
Si x 3 0 x 3 la ecuación no tiene solución.
Si x 3 0 x 3 la ecuación tiene dos soluciones:
x 3 2 x 1 x 4
x 3 2x 1
( x 3) 2 x 1 x 2 / 3
Sin embargo, estas dos soluciones no son aceptables porque contradicen la
hipótesis x 3 .
(b)
x 2 x 1 3 x 3 2 x 1
Si x 3 0 x 3 la ecuación no tiene solución.
Si x 3 0 x 3 la ecuación tiene dos soluciones:
x 3 2x 1 x 2
x 3 2x 1
( x 3) 2 x 1 x 4 / 3
Y en este caso ambas soluciones son aceptables pues ambas cumplen x 3
Las soluciones son dos: x 4 / 3 , x 2
10.1 F
Si x x y 10 y x y y 12 , determina x y .
10.2 F
Sea f ( x) x p x 15 x p 15 , con 0 p 15 . Determina el valor mínimo
tomado por f (x) para x en el intervalo p x 15 .
AIME 1983 #2
10.3 M
Determina, si es que existen, polinomios f (x) , g (x) y h(x) cumpliendo:
1 si x 1
f ( x) g ( x) h( x) 3x 2 si 1 x 0
2 x 2 si x 0
Putnam 1999
10.4 M
Determina el número de pares x, y que satisfacen el siguiente sistema de ecuaciones:
x 3 y 3
x y 1
10.6 F
Para cada terna a, b, c de números reales diferentes de cero, formamos el número
a b c abc
a b c abc
AHSME 1977 #8
11 Función "Parte entera".
Definición de función parte entera y función techo.
Definimos la “parte entera” (o "suelo") de un número real x , que denotaremos por x ,
como el mayor entero n menor o igual que x .
Observación: Definida así, “parte fraccional” equivale a tomar los decimales del
número y eliminar la parte entera solo cuando x 0 . Para números negativos ya no
podemos mantener esta idea. Por ejemplo:
x 3.7 x 4 x x x 3.7 (4) 0.3
Con Mathematica:
11.1 F
Determina el conjunto de soluciones de la ecuación x x 5 .
ASHME 1986 #7
11.2 F
Sea x un número real seleccionado aleatoriamente uniformemente entre 100 y 200. Si
x 12 , determina la probabilidad que 100 x 120 .
11.3 M
Determina el número de enteros 0 n 1000 que se pueden expresar de la forma
2 x 4 x 6 x 8x
AIME 1985 #10
11.4 F
Resuélvase la ecuación
x2 x 2 x
para x IR .
11.5 M
Resolver el sistema
x y z 200.0
x y z 190.1
x y z 178.8
Australia 1999
11.6 M
Determina la cantidad de números distintos que existen en la sucesión
12 2 2 20052
2005 , 2005 , …, 2005
11.7 M
Determina el entero positivo n para el cual
1
n
se acerca más a 123456789 .
ARML 2003
11.8 F
Supongamos que a es positivo, a 1 a 2 , y 2 a 2 3 . Determina el valor de
a12 144a 1 .
AIME 1997
11.9 F
Determina todas las soluciones reales de la ecuación 4 x 2 40x 51 0
11.10 F
Dado un número real a , sea a el mayor entero menor o igual que a . Determina la
región del plano coordenado de los puntos x, y tales que x y 25 .
2 2
11.11 MD
Determina todas las funciones f : IR IR que cumplen la igualdad
f x y f x f ( y)
Para todo x, y IR .
IMO 2010 #1
11.12 F
11.13 MD
n 1000
Determina el número de posibles enteros n que satisfacen
70
n
(A) 2 (B) 4 (C) 6 (D) 30 (E) 32
11.14 MD
p
El número a , donde p y q son enteros coprimos positivos, cumple la propiedad de
q
que la suma de todos los números reales x satisfaciendo
x x a x
2
es 420, donde x denota el mayor entero menor o igual que x y x x x denota la
parte fraccionaria de x. Determina p q .
(A) 245 (B) 593 (C) 929 (D) 1331 (E) 1332
11.15 F
Denotando por x el mayor entero menor o igual que x , Determina el número de
soluciones reales de la ecuación
(A) 197 (B) 198 (C) 199 (D) 200 (E) 201
12.1 D
5 tn 1 1
Definimos la siguiente secuencia recursiva: t1 20 , t2 21 , y tn .
25 tn 2
Determina t2020 .
AIME II 2020 #6
12.2 MF
Definimos la función f recursivamente por f (1) f (2) 1 y
f (n) f (n 1) f (n 2) n
(A) 2016 (B) 2017 (C) 2018 (D) 2019 (E) 2020
vale
a
a r
k 0
n
1 r
12.4 M
Un gusano se mueve durante todo el día y duerme durante toda la noche. Empieza en el
punto O y avanza una distancia de 5 unidades hacia el este. Cada noche, el gusano gira
60º en el sentido contrario al de las agujas del reloj, y al día siguiente avanza en esta
nueva dirección la mitad de la distancia del día anterior. Este gusano se acerca más y
más a un cierto punto P. Determina las coordenadas de dicho punto.
AIME I 2020 #8
12.5 M
La sucesión a1 , a2 , a3 ,... comienza con a1 49 . Para n 1, el término an1 se obtiene
añadiendo 1 a la suma de las cifras de an y elevando al cuadrado el resultado. Por
ejemplo, a2 4 9 1 196 . ¿Cuánto vale a2019 ?
2
12.6 F
n2
3n
12.8 M
Existe un único número real positivo x tal que los tres números
log8 (2 x) , log 4 x , log 2 x
en este orden, forman una progresión geométrica con razón positiva común.
Determina x.
AIME I 2020 #2
12.9 D
Existe una única secuencia de enteros no negativos a1 a2 ... ak tal que
2289 1 a1
2 2a2 ... 2ak
217 1
Determina el número k .
(A) 117 (B) 136 (C) 137 (D) 273 (E) 306
AMC 12A 2020 #19
12.10 F
¿Cuál es la mayor cantidad de números enteros consecutivos cuya suma sea 45?
12.11 M
3
Definimos recursivamente la siguiente sucesión de números por a1 1 , a2 ,y
7
an 2 an 1
an .
2an 2 an 1
p
Entonces a2019 se puede escribir como , con p y q enteros positivos coprimos.
q
Determina p q .
(A) 2020 (B) 4039 (C) 6057 (D) 6061 (E) 8078
12.13 MF
100 100
Determina (i j )
i 1 j 1
(A) 100100 (B) 500500 (C) 505000 (D) 1001000 (E) 1010000
12.14 M
Sea A el conjunto de todos los enteros cuyos únicos factores primos son 2, 3 y 5. La
suma infinita
1 1 1 1 1 1 1 1 1 1 1 1 1 1
...
1 2 3 4 5 6 8 9 10 12 15 16 18 20
m
de todos los recíprocos de los elementos de A se puede escribir como , donde m y n
n
son enteros positivos relativamente primos. Determina m n .
13.2 MF
Calcula
7! 6! 5!
7! 6! 5!
13.3 MF
Determina la x :
1 1 x
8! 9! 10!
13.4 MF
Dado un entero positivo n, representamos por n! el producto de todos los enteros de 1 a
n. (Por ejemplo, 6! 6 5 4 3 2 1 ) Determina el valor de N que satisface la siguiente
ecuación:
5! 6! 12 N!
1.1.2
6(1min 11seg ) 6 min 66seg 6 min 60seg 6seg 7 min 6seg (B)
1.1.3
1997 199819981998 1997 1998 108 104 1
1998 199719971997 1998 1997 108 104 1
Luego:
1997 199819981998 1998 199719971997
1997 1998 108 104 1 1998 1997 108 104 1 0
1.1.4
(A)
1.1.5
Aplicamos la propiedad distributiva:
2 2008 2008 8 2008 2 8 2008 10 20080 (C)
1.1.6
Probando vemos que es (A).
1.1.7
(D)
1.1.8
(A)
1.1.9
(B)
1.1.10
(B)
1.1.11
(C)
1.1.12
(E)
1.1.13
(E)
1.1.14
(D)
1.1.15
(A)
1.1.16
(E)
1.1.17
(A)
1.1.18
(A)
1.1.19
(E)
1.1.20
(C)
1.1.21
(C)
1.1.22
Aplicando la propiedad distributiva
8 4 8 3 8 2 8 1 8 4 3 2 1 8 10 80
1.1.23
60
1.1.24
44
1.1.25
400
1.1.26
0
1.1.27
80
1.1.28
44
1.1.29
800
1.1.30
9990
1.1.31
700
1.1.32
400
1.1.33
145
1.1.34
2000
1.1.35
4000
1.1.36
4545
1.1.38
404
1.1.39
3765
1.1.40
212
1.1.41
10
1.1.42
0
1.1.43
44
1.1.44
20190
1.1.45
400
1.1.46
2000
1.1.47
36
1.1.48
300
1.1.49
50
1.1.50
1.1.51
-10
1.1.52
1365
1.1.53
1300
1.1.54
A 42
C B5
B A / 2 B 21 C 21 5 16
(B)
1.2.1
3.14 3 9.42 y los valores de x pedidos son -9, -8, -7, ..., 7, 8, 9. Un total de 19 (D).
1.2.2
(E)
1.2.3
(E)
1.2.4
1.2.5
(B)
1.2.6
(E)
1.2.7
a 11 , b 5 , c 1 , a b c 55
1.3.1
(A)
1.3.2
(D)
1.3.3
(B)
1.3.4
1 2 3 4 5 6 7 8 1 2 3 4 5 6 7 8
2 2 5 2 7 8 2240 ( D)
1 2 3 4 5 6 7 8 36
1.3.5
(A)
1.3.6
(A)
1.3.7
(E)
1.3.8
(C)
1.3.9
(D)
1.3.10
(E)
1.3.11
(C)
1.3.12
(D)
1.3.13
(E)
1.3.14
(E)
1.3.15
(E)
1.3.16
(C)
1.3.17
(D)
1.3.18
(C)
1.3.19
(C)
1.3.20
(B)
1.3.21
(A)
1.3.22
(C)
1.3.23
(C)
1.3.24
(C)
1.3.25
(D)
1.3.26
(D)
1.3.27
(A)
1.3.28
(A)
1.3.29
(D)
1.3.30
3333 6666 1111 6 1111 1111 1111 1111
3 3 2 5 5 11 55 (D)
101 303 101 3 101 101 101 101
1.3.31
20042 2 2004 20042004 2 2 10022004 2
22004 2 4012 (C)
1002 1002 1002
1.4.1
100
1
0.5 100
4
50
450
1 1 1
450 250 450 50 450 1 (C)
2
100
2 2 4
Fuente: Simple Math en Facebook
1.4.2
(C)
1.4.3
(D)
1.5.1
1 1 1 30 1 3 1
(100% 70%) 30% 10% . Queda 70% 10% 20% (C)
3 3 3 100 3 10 10
1.5.2
150 3
a b b 2a 3b 3b es el doble de a, es decir, un 200 % (D).
100 2
1.5.3
36 72 36 100
Planteamos la ecuación 100 x 50 x 50 (D)
100 x 100 72
1.5.4
Basta analizar la figura. (A)
1.6.1
4 2 16 21
4 21 5 16 20 4 20 21 20 5 15
5 25 21
2
4 21 5 24 20 4 20 21 20 5 25
Luego son los enteros 16, 17, 18, 19, 20, 21, 22, 23, 24, hay 9 en total (A).
1.6.2
5
Dividiendo numerador entre denominador vemos que 1.666... , y sabemos que
3
3.14 2 6.28 , luego los enteros del intervalo serán 2, 3, 4, 5 y 6, en total 5 (D)
1.6.3
1 1 1
257 2 5 7 , y por tanto sí pertenece a dicho intervalo.
2 5 7
1.6.4
28 1 a 3 218 1 3 28 1 a 3 218 1
Por un lado,
63 216 257
3
2 1 257 3
8 3
7 343 257
Luego el cubo más pequeño buscado es 7 3 .
Determinar una aproximación de 3
218 1 es más difícil, pero observamos que
64 26 3 218 3 218 1 y está claro que será la raíz cúbica entera más grande que podremos
encontrar, luego el total de cubos será 64 7 1 58 (E).
1.7.1
2 horas a 60 millas por hora = 120 millas = 4·30 millas = 4 galones de gasolina = 8$ en
gasolina = 4$/hora
120 millas·0.50$=60$, 60$/2horas=30$/hora.
30$/hora-4$/hora=26$/hora de beneficio. (E)
1.7.2
Sean A el número de partidos disputados por el equipo A y sea B el número de partidos
disputados por el equipo B.
5 2
8 B 3 A 7 15B 16 A 7 24
A 42 , B 56 (C )
3 1
B A7 9 B 8 A 7 24
8 3
1.7.3
A1 42
A A2 16 9 7 7
2
N 1 0.77 78 ( E )
A2 3 A2 9 9 9
1.7.4
Vemos que la ecuación que relaciona los volúmenes de los recipientes es
5 3 3 6 33 9 A 9
A B A B B B
6 4 45 25 10 B 10
1.7.5
En la primera media hora:
espacio espacio
60 espacio 60 0.5 30 millas en la primera media hora.
tiempo 0.5
En la segunda media hora:
espacio espacio
65 espacio 65 0.5 32.5 millas en la segunda media hora.
tiempo 0.5
Luego en la tercera media hora tiene que recorrer 96 30 32.5 33.5 millas, y por tanto su
velocidad media será
espacio 33.5
velocidad 67 millas/hora (D)
tiempo 0.5
1.7.6
4 w 3 x 8w 6 x 8w 1z 16w 2 z
8w 1z , 16w 3 y 16 : 3 (E)
1z 6 x 3y 2z
1.7.7
Paul corre cuatro veces la velocidad de Eve, luego recorrerá cuatro veces la distancia de Eve,
digamos 4p y 1p. En el momento en que se encuentran Paul y Eve, Ina habrá recorrido la mitad
de camino que Paul, es decir, 2p, y Paul tendrá que recorrer 4p, y Ina 2p, luego en total, los
1800 metros se descomponen en
1800 2 p 2 p 4 p p 9 p p 1800 / 9 200m
Y por tanto de A a B hay 4p, es decir, 800 m.
1.7.8
El producto de dos números muy próximos a 1 está muy próximo a 1, y al multiplicar por un
número menor que 1 el resultado es menor que 1, luego la solución más plausible es q. (B)
1.7.9
Sea g el peso de un dromedario y e el peso de un elefante. Tenemos Q G g
Q
M
gM G QM
g M E e e (D)
E E GE
1.7.10
Se cumple 3x 3x 4x 180 10x 180 x 18 , luego el ángulo mayor mide 4 18 72º
(D)
1.7.11
Sean a1 , a2 , v1 , v2 el número de canicas azules y verdes en cada bolsa.
v1 v2 95
a1 v1 a2 v2 9v1 v1 8v2 v2 10v1 9v2
Este sistema se resuelve para v1 45 , v2 50 , luego a1 45 a2 50 a1 a2 5 (A)
1.7.12
(B)
1.7.13
3 p 2m
9 p 6m 4n 18 p 8n (B)
6m 4n
2.1.1
(E)
2.1.2
(B)
2.1.3
(C)
2.1.4
(E)
2.1.5
(A)
2.1.6
(E)
2.1.7
(C)
2.1.8
(A)
2.1.9
(C)
2.1.10
(A)
2.1.11
101/ 20
2.1.12
3/2
2.1.13
(B)
2.1.14
(A)
2.1.15
(B)
2.1.16
Aplicando la igualdad “Diferencia de cuadrados”:
100 2 7 2 (100 7)(100 7)
70 2 112 (70 11)(70 11)
Está claro que el resultado es 1 (A)
2.1.17
Vemos que se cancelan por parejas, y por tanto el resultado es 0 (C)
2.1.18
1010 2 2020 2 3030 2 1010 2 2 1010 3 1010 1010 2 1 4 9
2 2
2020 2 1010 2 1010
1010 2 14
7070
2 1010
(E)
2.1.19
342 3237 33
2 7
2714
3 4
42 28
4
27
428 4227 2
16
14
2.1.20
2255 22511 225
11
55 5 11 5 11 25 11
2 2 2 2 2
2.1.21
20212 1 20212 12 2021 12021 1 2021 1 2020
2022
2020 2020 2020 2020
2.1.22
33 33 33 3 33 34 (A)
2.1.23
15 12 1 12 1 1 0 (C)
2 5 5
25
2.1.24
66 66 66 66 66 66 6 66 67 (B)
2.1.25
1530 3 5 330 530 330 530 530
30
15 515 (E)
4515
3 5
2
15 15
3 5
2
15 3 5
30
15
5
2.1.26
(D)
2.1.27
(C)
2.1.28
(E)
2.1.29
(E)
2.1.30
Los podemos escribir con exponente 8:
108 108
512 512/ 8 5
8 3/ 2 8
5 125
3
8 8
2 8
8
2 24 238 3 8
2.1.31
2n 4 2 2n 2n 4 2n 1 2n 1 23 1 23 1 7
3 (D)
2 2n 3 2n 4 2n 4 2 8
2.1.32
10002 10002 10002 10002 1000
500 (C)
2522 2482 252 248252 248 4 500 2000 2
2.1.33
(C)
2.1.34
k 21995 21998 21997 21996 21995 21995 23 22 21 1 21995 3 k 3
2.1.35
25012 2500 2 (2501 2500)(2501 2500) 1 5001 5001
2.1.36
23 4 n 23 2 2 n
2 3 2 2 n 2 2 n 3
2n 3 15 n 6
85 23 5
215
2.1.37
Aplicando la identidad “suma por diferencia”, la solución es 55
2.1.38
(D)
2.1.39
(C)
2.1.40
490 489 489 489 489 490 4 489 490 490 0
2.1.41
(B)
2.1.42
2625
2.1.43
555
2.1.44
5627 5627 5637 5617 5627 2 5627 105627 10 5627 2 5627 2 10 2
5627 5627 10 10 100
2 2 2 2
2.1.45
2427 827
24 8 327 27
327
327
34
911 911 911 3 911 3 32
11
33 22
323
2.1.46
n 1 n 1 n 1 n 12 1 (n 1) 2 n 2 2(n 1)n 1
(n 1) 2 n 2 1 2(n 1)n
(n 1) 2 n 2 n 2 (n 1) 2 1 2(n 1)n n 2 (n 1) 2 (n 1)n 1
2
(n 1) 2 n 2 n 2 (n 1) 2 (n 1)n 1
(n 1) 2 n 2 n 2 (n 1) 2 (n 1)n 1 1
1
n(n 1) n(n 1) n(n 1)
(n 1) 2 n 2 n 2 (n 1) 2 1
1
n(n 1) n(n 1)
(n 1) 2 n 2 n 2 (n 1) 2 1
1
n (n 1)
2 2
n(n 1)
1 1 1
1 1
n (n 1)
2 2
n(n 1)
1
n(n 1)
1 1
1 2 1
n (n 1) 2
2.1.47
2
5 3
5 3
x
2 x 5 3 5 3 52 3 25 3 22 x 11
2
2.1.48
7202 1802
720 180720 180 540 900
900 30
540 540 540
2.1.49
(A)
2.1.50
10 10
10 x 100 2 x 10005 10 x 10 2
2x 3 5 x
104 x 1015
10 4 x x 1015 105 x 1015 5 x 15 x 3 (C )
2.2.1
(C)
2.2.2
(D)
2.2.3
(B)
2.2.4
(C)
2.2.5
(C)
2.2.6
2 0
1 52 0 1
5 1 1 52 0
1
5
1
52
5
1
5
(C)
2.2.7
2 2 2
2 1 1 8 1 7
2 (C)
22
4 4 4 4
2.2.8
(B)
2.2.9
8 5
45
2
23 2 2
5 2 5
2
15
210
2
25
2
210
2.3.1
(D)
2.3.2
2.3.3
(D)
2.3.4
(A)
2.3.5
Está claro que x 0 es solución de esta ecuación. Y es la única, puesto que la función
a bx b cx c ax es creciente y la función b ax c bx a cx es
decreciente.
2.3.6
Primera versión.
3
1729 x 3 x 19 3 1729 x 19 3 x
3
2
1729 x 19 3 x 193 3 192 2 3 x 3 19 3 x x 6859 1083 3 x 57 3 x x
2
0 5130 1083 3 x 57 3 x 2
Con el cambio de variable u 3 x solo nos queda resolver la ecuación de segundo grado
u 10
0 5130 1083u 57u 2 0 90 19u u 2 0 (u 10)(u 9)
u 9
10 u 3 x x 1000
9 u 3 x x 729
Segunda versión.
Llamando a 3 1729 x y b 3 x , tenemos que resolver el sistema
a b 19
3
a b3 1729
2.3.7
Sean a 4 97 x y b 4 x . Queremos solucionar el sistema
a b 5
4
a b 97
4
(a b) 4 a 4 4a 3b 6a 2b 2 4ab 2 b 4 a 4 b 4 ab 4a 2 6ab 4b 2
4
2 2
a b ab 4a 6ab 4b a b ab (2a 2b) 2ab
4 4 4
2
a b
4 4
ab4(a b) 2
2ab a 4
b 4
2ab2(a b) 2
ab
En nuestro caso:
54 97 2ab 2 52 ab 97 2ab50 ab
a 4 97 x 3 97 x 34 x 97 34 16
x 16
b 4 x 2 x 24 16
2.3.8
Tenemos 3 4 6 13 segmentos de 1 unidad y 2 2 0 4 segmentos de longitud 2,
luego en total, 13 4 2 (C)
2.3.9
1 1 3 1 3 5 1 3 5 7 1 2 3 4 10 (C)
2.3.10
Pasando a forma exponencial tenemos
36
N 25 N 25 / 36
a
N b N c N a N b N N 1 / c a N b N 1 / c 1 a N N 1 / c 1 / b a N 1 / c 1 / b 1 N 1 / c 1 / b 1 / a
El sistema
1 c bc 25 c(b 1) 24 23 3
abc 36 abc 36
2.3.11
Elevamos al cuadrado:
x 3 2 2 32 2
2 2 2
x 2 3 2 2 3 2 2 3 2 2 3 2 2 2 3 2 2 3 2 2
3 2 2 3 2 2 2 3 2 2 3 2 2 6 2 32 2 2 2
6 2 32 4 2 6 2 1 6 2 4
Y por tanto x 4 2 (A)
2.3.12
Resolveremos este problema como aplicación directa de la identidad suma por diferencia:
49 x 2 25 x 2 3
3 49 x 2 25 x 2 49 x 2
25 x 2 49 x 2
25 x 2
2
49 x 25 x 49 x 25 x 24
2
2 2
24
49 x 2 25 x 2 8 (A)
3
2.3.13
Sea a 16 5 .
Entonces
5 1 4
5 1 8
5 1
16
5 1 a8 1 a 4 1 a 2 1 a 1
a a a ... a 1
15 14 13
a15 a14 a13 ... a 5
a16 a14 a13 ... a
a a15 a14 a13 ... 1
a
a15 a14 a13 ... a 1 a15 a14 a13 ... a 1 a15 a14 a13 ... a 1
y por tanto:
48
4
1 a 48 a16 53 125
3
5 1 4
5 1 8
5 1 5 1
16
Observación. Tal vez, un razonamiento más elegante sería tener en cuenta que
a16 1
a15 a14 a13 ... a 1
a 1
2.3.14
34 37
34 1 33
34 28
34 4 32 2 18
7 7 7
2.3.15
53 10 6 53 2 5 3 2 52 2 3 2 5 3 2
5 2 3 2 5
2 2
3 2 5 2 3
2
Observamos que 1.42 1.96 , luego 2 está entre 1.4 y 1.5. Por otro lado, 1.72 2.89 , luego
3 está entre 1.7 y 1.8. Así pues, 5 2 3 , y por tanto
53 10 6 5 2 3
2
5 2 3
2.3.16
810 410 23 2 2 2 2 2 1
10 2 10 30 20 20 10
84 411 2 2 2 2 2 1 2
3 4 2 11 12 22 12 10
810 410 2 2 1 2
20 10 20
2 8
84 411 2 1 2 2
12 10 12
Luego
810 410
28 28 / 2 24 16 (B)
8 4
4 11
2.3.17
5 2 1
2 1
5 2 1 2 1
5 2 1
2 2
52 1 5
1
5 5 5 5 5
2.3.18
(C)
2.3.19
(C)
2.3.20
2.3.21
2.3.22
50 32 128 5 2 4 2 8 2 2
x x
Luego 2 2 x / 2 2 1 x 2
2
2.3.23
Pasamos a índice común:
4
5 12 53 12 125
3
4 12 44 12 44 12 256
Luego:
4
5 12 125 12 256 3 4
2.3.24
(B)
2.3.25
Los cuadrados exactos que pueden aparecer son los siguientes 13:
02 0 , 12 1 , 22 4 ,…, 122 144
y cada uno de estos valores son posibles, tomando x entre 0 03 y 1443 :
122 144 3 0 ,…, 02 144 3 1443
La respuesta correcta es 13.
2.3.26
(B)
2.3.27
502 142 50 1450 14 36 64 36 64 6 8 48
2.3.28
217 219
217 1 2 2
217 5
217
216 216/ 2 28
10 10 10 2
2.3.29
3 52 2 4 3 5 3 4 3 3
1
3 3 3
2.3.30
72 32 2332 25 6 2 4 2 10 2
1
128 8 2 7 23 8 2 2 2 10 2
2.3.31
98 32 98 32 2
2 2 2
98 32 98 32 2 2 7 2 32
130 2 7 2 82 130 112 18
2.3.32
522 482
52 4852 48 100 4
100 4 10 2 20
1
2 5 2
45 45 45 4 5 20
2.3.33
3
25 2 3 1 x 3 2 3 1 x 2 3 1 x 2
1 x 23 8 x 8 1 7
2.3.34
1 3 1 3 1
3 1
3 1 3 1 2
1 5 3 5 3
5 3 5 3 5 3 2
1 7 5 7 5
7 5 7 5 7 5 2
1 9 7 9 7
9 7 9 7 9 7 2
Y por tanto
1 1 1 1
1 3 3 5 5 7 7 9
1
2
3 1 5 3 7 5 9 7
1
2
9 1 1
2
3 1 1 ( A)
2.3.35
95 38
3
2 4
38
310 38
38 32 1
34 32 1
9 5 34 3
2 5
34 310 34 34 36 1
36 1
32 1 10 1 9
32 32 32
3 1
6
730 73 73
2.3.36
1222 1212 122 121122 121 243 1 243 35 , y por tanto la solución es n 5 .
2.3.37
7 5 7 5 7 5 2 2
49 5
44
44 44
44 44
44 2 11
44 44 44 44 44 44 44
3.1.1
4x 6x 9x
x x x x
4x 6x 9x 6 9 3 9
4x 6x 9x x x 1 1
4 4 2 4
x x x
4 4 4 4 4
x 2
9 3 3 3
x 2 2 x x
Observamos que 2
4 2 2 2
x
3
Luego, con el cambio de variable u llegamos a la ecuación cuadrática 1 u u 2
2
1 12 4 1 (1) 1 5
1 u u u u 1 0 u
2 2
2 2
La segunda ecuación
3 1 5
x
2 2
1 5
no tiene solución puesto que 0.618 0 , es negativo.
2
3.1.2
9 x 2 240 9 x 9 x 2 9 x 240 9 x 9 2 1 240
240
9 x 80 240 9 x 3 32 x 3 2 x 1 x 1 / 2
80
3.1.3
7 8x 27 x
x
23 33
x
x
23 33
x
2 x 3x
x
3 3
7
u 3 v3
u v u 2 uv v 2
u 2 uv v 2 7
uv u 2 uv v 2
6 uvu v uvu v uv 6
0 2u 3v
0 u 2 uv v 2 uv u 2 uv v 2 2u 3v 3u 2v
7 13 1
6 6 6 0 3u 2v
x
2 v 3x 3
0 2u 3v 2u 3v x 1
3 u 2x 2
x
3 v 3x 3
0 3u 2v 3u 2v x x 1
2 u 2 2
3.1.4
Se puede comprobar fácilmente (!!!!) que x 2 es solución de la ecuación. Veamos que es la
única solución. En efecto, dividiendo entre 13x ambos lados de la ecuación llegamos a
x x x x
10 11 12 14
1
13 13 13 13
La parte de la izquierda es una función decreciente, mientras que la parte de la derecha es una
función decreciente, por lo tanto tendrán como mucho un punto de corte.
Nota: En general,
a 2 (a 1)2 ... (a k )2 (a k 1)2 (a k 2)2 ... (a 2k )2
para todo a k (2k 1) , k IN .
Fuente de esta solución: 101 Problems in Algebra from the training of the USA IMO team (Adreescu, Feng, 2001), página 38.
3.1.5
Antes de elevar al cuadrado verificamos que el interior de la raíz debe ser un valor no negativo:
x x
6x 6 2
6 9 0 6 9 x 1 1 1 x 0
x x x x
9 9 3
x
2
En donde hemos aplicado que f ( x) es una función decreciente y f (0) 1 .
3
Supongamos que x 0 .
Ahora pasamos a eliminar el radical elevando al cuadrado los dos lados de la ecuación:
6 9 2 3 3 3 3 2 3
2 x 3x 6 x 9 x 2 x 3x
2 x x x x x x x
3 3
2
Que es una solución aceptable porque 1 log 2 / 3 (2) 0
3
3.1.6
Aplicando la identidad
log a c
log a b log b c log a c log b c
log a b
Tenemos:
1 1 log 2 2
2 a 42 a log 2 42 log 42 2
a log 2 42 log 2 42
1 1
De la misma forma log 42 3 y log 42 7 , y por tanto:
b c
log 42 2 log 42 3 log 42 7 log 42 2 3 7 log 42 42 1
1 1 1
a b c
Fuente del enunciado: SyberMath en Youtube
3.1.7
33 2 x 5 2 x 2 x 25 2 x 2 x 25 1 2 x 33 1 33
33
2 x x 0 (C)
Fuente: Simple Math en Facebook
3.1.8
95 x1 32
5 x 1
32(5 x1) 310x2
10 x 2 12 10 x 10 x 1
27 4 3
3 4
3 12
3.1.9
84 88 812 23 12
236
36 9 x x 4
512 x 29 x
29 x
3.1.10
2 x 4 y 1 2 2
y 1
2 2 y 2 x 2 y 2
3
y
3x1 27 y 3
33 y x 1 3 y 2 y 2 1 3 y 3 y x 2 3 2 8
Por lo tanto la respuesta corrcta es x y 8 3 11 (D)
3.1.11
(C)
3.1.12
x9
3.1.13
21994 4997 8665 21994 2 2 997
23
665
21994 2 2997 23665
21994 21994 21995 219941 1 2 21994 4 219942 21996 2 4499 16 499
y la respuesta correcta es (C).
3.1.14
No tiene ninguna solución (B)
3.1.15
32011 9 n 9 n 9 n 3 9 n 3 32 n
32n1 2011 2n 1 n 1005 (A)
3.1.16
2 x7 2 2 x2 2 x7 2 x3 x3 2 1
4
16 1 15
x3
x3
2 2 x3 2 x3 3 2 x3
5 2 5 2 5 5 5
3.1.17
3x 3x 3x 3 3x 3x1 81 34 x 1 4 x 3
3.1.18
3a 81b2 34 b 2
34(b2) a 4(b 2)
5
b
5a3 125b 3
53b a 3 3b
a 4(b 2) a 12
ab 60
a 3 3b b 5
3.2.1
Basta aplicar la fórmula del cambio de base:
log b log c log d log a
log a b log b c log c d log d a 1
log a log b log c log d
3.2.2
log10 a 2 15a 2
a 2 15a 10 2 100
100 (20)(5)
a 2 15a 100 0
15 20 5
a 20
(a 20)(a 5) 0
a 5
Efectivamente,
a 20 a 2 15a 202 15 20 400 300 100 102
a 5 a 2 15a (5)2 15 (5) 25 75 100 102
3.2.3
Consultar PR/1.11
3.2.4
Primera versión.
luego
log 2 x3 32020 log 2 x3 320 101
101 log 2 x3 320
Y por tanto
log a c
Ahora aplicamos la fórmula del cambio de base: log a b
log b c
Y por tanto:
log 2 x 320 x3
101 20
log 2 x 2 x 3
log 2 x3 3 x
Segunda versión.
m
Utilizando la identidad log a n b m log a b
n
3.2.5
ln a 2 b 2 2 ln a b ln a 2 b 2 ln a b ln
2 a 2 b2
2
a b
(a b)(a b) a b a b
ln ln ln
1
ln 0.693
a b ab 2(a b) 2
2
La solución es (C).
3.2.6
log c b log 4 x log 4 x
De la igualdad log a b se deduce log 2 x log 2 x 2 log 4 x
log c a log 4 2 1 / 2
1
(En general se puede deducir la igualdad log a b c log a c )
b
Luego
De nuevo
log 4 n log 4 n
log16 n
log 4 16 2
Luego
log 4 n
(*)
2
log 4 n log n log n2 4 log n
2
log 4 n 4 4 4
2 4
log 4 n 4 log 4 n 4 n 4 4 256 2 5 6 13 ( E )
3.2.7
Primera versión.
log 2 6 log 3 6 log 2 2 3 log 3 2 3 log 2 2 log 2 3 log 3 2 log 3 3
1 log 2 3 log 3 2 1 log 2 3 log 3 2 2 log 2 3 log 3 2 2 log 2 3 log 3 2
1 2x x 2 1 ( x 1) 2 x 1 x 1 1
2 x x
x x x x x x x
3.2.8
BCC ' B' 1 ln( x 2) ln( x 1) ln( x 1) ln( x) 1 ln x 2 ln x 1
2 2 x 1 x
CDD' C ' 1 ln( x 3) ln( x 2) ln( x 2) ln( x) 1 ln x 3 ln x 2
2 2 x2 x
ABCD ABB ' BCC ' B' CDD' C ' ADD'
1 x 1 1 x 2 x 1 1 x 3 x 2 3 x 3
ln ln ln ln ln ln
2 x 2 x 1 x 2 x2 x 2 x
x 1 x2 x 1 x3 x2 x 3
2ABCD ln ln 2 ln ln 2 ln 3 ln
x x 1 x x2 x x
x 1 x 2 x 1 2 x 3 x 2 2
ln 3 ln x 3
x x 1 x x 2 x x
( x 1) 2 ( x 2) 2 ( x 3) x 33
ln ln
3 x3
x
( x 1) 2 ( x 2) 2 ( x 3) x3 ( x 1) 2 ( x 2) 2 ( x 1)( x 2)
ln
3
ln 2 ln
x 3
x 3 x 3 2
x3
( x 1)( x 2) 91 ( x 1)( x 2) 91 7 13
ln ln
x3 90 x3 90 90
( x 1)( x 2) 91 15
x
x3 90 12
Descartando el valor negativo, llegamos finalmente a x 12 ( D) .
3.2.9
log a c
Utilizando la fórmula del cambio de base: log b c
log a b
log log x
log x log x
54 log log x ( x)
log log x 54
3 log
x log x 56 log x log x
56
3
56
3
log x log x log x log log x log x
log x
54
1 log x 1 1 14
log x log x log x
2 54 2 54 27
56 27
log x 36
3 14
b 2 / 3 36 b 363 / 2 216
3.2.10
2
x
log 2 log 2 x log 2 y
2
y
Y aprovechamos la identidad A B A B 4 AB
2 2
Finalmente:
2
x
log 2 log 2 x log 2 y 2 log 2 x log 2 y 2 4 log 2 x log 2 y
y
6 4 4 36 16 20
2
3.2.11
Las tres desigualdades que se deben cumplir son:
log 2 x log 4 x 3 , log 2 x 3 log 4 x , log 4 x 3 log 2 x
Así pues tenemos 1 log 4 x 3 41 4log4 x 43 4 x 64 , así pues, los valores enteros
posibles van del 5 al 63, es decir, 59 (B).
3.2.12
Analizamos la operación binaria:
an n(n 1)an 1 n1 / log7 ( n 1) an 1 n1 / log7 ( n 1)
log7 ( a n1 )
nlog7 ( an1 ) / log7 ( n 1)
Y por tanto
log n an log n 1 (an 1 )
Así pues, log 2019a2019 log 2018(a2018) log 2017(a2017) ... log 3 (a3 )
Así pues,
log 2019a2019
1
log 2 (7)
log 7 (2)
3.2.13
Aplicando la “regla de la cadena” de los logaritmos:
log 3 7 log 7 11 log1115 log15 19 log19 23 log 23 27 log 3 27 log 3 33 3
log 5 9 log 9 13 log1317 log17 21 log 21 25 log 5 25 log 5 52 2
Y puesto que hay 11 factores, ya los hemos utilizado todos, y el resultado es
3 2 6 (C)
3.2.14
log a c
Aplicando la identidad del cambio de base: log a b log b c log a c log a b
log b c
Tenemos:
log 3 x 4 log 2 x 8 log 3 x 22 log 2 x 23 2 log 3 x 2 3 log 2 x 2
2 log 2 x 2
log 2 x 3x
3 log 3 x 2
2 x 2 / 3 3x 2 x 2 3x 3 4 x 2 33 x3 4 33 x x 4
27
Y la respuesta correcta es 4 27 31 (D)
3.2.15
De la identidad log a b log b c log a c se deduce
log a c
log a a 2 log a 2 c log a c 2 log a 2 c log a c log a 2 c
2
Y también a la segunda:
log 3 3x y log 9 3x 2 4 xy Ky 2 log 32 3x 2 4 xy Ky 2
log 3 3x 2 4 xy Ky 2
2 log 3 3x y log 3 3x 2 4 xy Ky 2
2
2
log 3 3x y log 3 3x 2 4 xy Ky 2 3x y 3x 2 4 xy Ky 2
2
16 7 K K 16 7 9
3.2.16
Primera versión.
Aplicando las identidades básicas de los logaritmos:
log 2 80 log 2 160 log 2 80 log 20 2 log 2 160 log 40 2
log 40 2 log 20 2 log 40 2 log 20 2
log 80 log 160
log 20 80 log 40 160 log 20 log 40 log 40 log 20 log 80 log 160
log 40 2 log 20 2 log 2 log 2
log 2
2 log 20 log 40
log 40 log 20
log 40 log 20 log 80 log 40 log 160 log 20 log 80 log 40 log 160 log 20
(*)
log 2 2 log 20 log 40 log 2 2
Segunda versión.
Se podría desarrollar un razonamiento más elegante aplicando las identidades
1 1
log 40 2 , log 20 2
log 2 40 log 2 20
2
Fuente de esta segunda versión: [Link]
3.2.17
log 5 7 log 3 5 log 3 5 log 5 7 log 3 7 , luego:
3log5 7log3 5 3log3 7 7
3.2.18
(C)
3.2.19
1 log x y 3 log x log y 3 log x 3 log y
1 log x 2 y log x 2 log y 2 log x log y
2 1
Y resolviendo este sistema llegamos a log x , log y .
5 5
3
Por lo tanto: log xy log x log y (D).
5
3.2.20
Mediante logaritmos:
x 2
4x 5
x 2 x 30
1 log x 2 4 x 5
x 2 x 30
log 1 0
x x 30 0 (a)
2
x 2 x 30 log x 2 4 x 5 0
log x 4 x 5 0 x 4 x 5 1 (b)
2 2
x 6
(a) 0 x 2 x 30 ( x 6)( x 5)
x 5
(b) x 4 x 5 1 x 4 x 4 0 0 ( x 2)( x 2) x 2
2 2
3.2.21
2 x 15 x log 2 15
15 y 32 25 log 2 15 y log 2 25 5 y log 2 15 5 y
5
log 2 15
Luego:
5
xy log 2 15 5 (A)
log 2 15
3.2.22
4x 6x 9x
x x
4x 6x 9x 6 9
4x 6x 9x x x 1
4 4
x x x
4 4 4 4 4
2
3 x
x
32
x
3
1 2
2 2 2
x
3
Mediante un cambio de variable t , la ecuación se transforma en
2
1 5
1 t t 2 t 2 t 1 0 t
2
1 5
x
3
Observamos que 0 no es aceptable porque una exponencial no puede ser
2 2
negativa, luego la única solución aceptable es
3
x
1 5 1 5
x log 3 / 2
2 2 2
3.2.23
Primera versión.
Aplicamos la fórmula del cambio de variable:
log 22 x log 202 x
k log 20x 22 x log 2 x 202 x k
log 20 x log 2 x
log 22 log x log 202 log x
k
log 20 log x log 2 log x
a c a ac
b d b bd
Luego:
log 202 log 22 log 202 / 22 log 101 / 11 log 101 / 11
k
log 2 log 20 log 2 / 20 log 1 / 10 1
log 101 / 11 log 11 / 101
Segunda versión.
20 x 22 x
k
10 2 x 22 x
k k
11 11
10k 202 x 22 x 10k k log
2 x k
202 x 101 101
4.1.1
2 f (3) a 3 b 3 3 5 a3 b3 3 5
4 2 4 2
5 2 3 a34 b32 5
f (3) a 34 b 32 3 5 a 34 b 32 5 3 5 2 3 (D)
4.1.2
7 f (7) a (7)7 b (7)3 c (7) 5 a (7)7 b (7)3 c (7) 7 5 12
a 77 b 73 c7 12 a 77 b 73 c7 12 a 77 b 73 c7 12
Y por tanto:
f (7) a77 b73 c7 5 12 5 17 (A)
4.1.3
Basta evaluar este polinomio para x 1 , es decir:
a7 a6 ... a0 a717 a616 ... a0 3 1 1 27 128 (E)
7
4.1.4
Una suma de cuadrados solo puede ser cero cuando todos sus elementos son cero, luego la
única solución posible es x y z 0 (B)
4.1.5
k h 1
x 2 k x 7 x 1x h x 2 hx x h x 2 h 1x h k 8 (C)
h 7
4.1.6
x 2 3x 1 0 x 2 3 x 1
x3 8 x x x 2 8 x x3x 1 8 x 3x 2 x 8 x 3x 2 9 x
3(3x 1) 9 x 9 x 3 9 x 3 ( B)
4.1.7
Sea esta raíz común. Entonces, sustituyendo en la ecuaciones y restando una de la otra:
1988 2 a 8891 0
6903 6903 0 1 1
2 2
8891 a 1988 0
2
Y ahora, sustituyendo en cualquiera de ellas:
1 1988 12 a 1 8891 0 a 8891 1988 10897
1 1988 12 a (1) 8891 0 a 8891 1988 10897
Y por tanto los posibles valores de a son 10897 y -10897.
4.1.8
42
42 x 2 y 2 ( x y)( x y) ( x y) 3 x y 12 2 x 2 y 2( x y) 24
3
2 x 2 y 242 576
2
4.1.9
Px 2 x 2 b1 x c1 , Qx 2 x 2 b2 x c2 .
54 2 162 b1 16 c1
2 54 16b1 b2 c1 c2
54 2 16 b2 16 c2
2
53 2 20 b1 20 c1
2
2 53 20b1 b2 c1 c2
53 2 20 b2 20 c2
2
Con el cambio de variable x b1 b2 , y c1 c2 , obtenemos el sistema
2 54 16 x y
2 54 16 x 2 53 20 x 2 54 2 53 20 x 16 x
2 53 20 x y
1 1
254 53 4 x 1 2 x x y 2 54 16 2 54 8 116
2 2
4.2.1
x 4 4 x 4 4 x 2 4 4 x 2 ( x 2) 2 4 x 2 ( x 2) 2 2 x x 2 2 x x 2 2 x
2
x 2 x 2x 2 x 2
4.2.2
Puesto que n es negativo, y el polinomio es mónico, tendremos que
x 2 x n ( x a)( x b)
para ciertos enteros a, b 0
Luego
1 b a b 1 a
x 2 x n ( x a)( x b) x 2 (b a) x ab
n ab n ab
Luego 1 ab a(1 a) 100
a 1 b 2 ab 2
a 2 b 3 ab 6
a 3 b 4 ab 12
a 4 b 5 ab 20
a 5 b 6 ab 30
a 6 b 7 ab 42
a 8 b 9 ab 72
a 9 b 10 ab 90
4.2.3
3x 24 x 1 3x 24 x 1 3x 24 x 1 4 x 1 3x 21 1
3x 2 1 3x 1
Si x 4 3x 3 11 (D)
4.2.4
1 1 1 r r r r r r
3 1 2 1 2 3
r1 r2 r2 r3 r3 r1 r1 r2 r3 r1 r2 r3 r1 r2 r3 r1 r2 r3
b c d
f ( x) ax3 bx 2 cx d x3 x 2 x
a a a
Aplicando las fórmulas de Vieta:
d
r1r2 r3
a
b
(r1 r2 r3 )
a
r r r b/a b
Por lo tanto, 1 2 3 .
r1 r2 r3 d /a d
f (0) a03 b02 c0 d d
3 2
1 1 1 1 a b c
f a b c d d
2 2 2 2 8 4 2
a b c
3 2
1 1 1 1
f a b c d d
2 2 2 2 8 4 2
1 1 a b c a b c b b
f f d d 2 2d 2d
2 2 8 4 2 8 4 2 4 2
1 1
f f b 2d b 4d
b 4d
997 2 2
2 b 4d b
2 2
f (0) d d 2d 2d 2d 2d
b b
995 997 2 2 995 1990
2d d
Luego la solución es 1990.
4.2.5
La ecuación de la circunferencia con centro (c, c) será de la forma
( x c) 2 ( y c) 2 r 2
y sustituyendo en (*)
(*) 2109 13 r 2
192 r 2
4.2.6
Vemos que f (0) 2 d 2 y que f (1) f (1) 0
0 f (1) a (1)3 b (1)2 c (1) d a b c 2
0 f (1) a (1)3 b (1) 2 c (1) d a b c 2
Sumando estas dos igualdades obtenemos 0 0 0 2b 4 b 2
4.2.7
Factorizando el polinomio x 2 3x 2 vemos que tiene raíces 1 y 2. Luego el polinomio
x 2 5x k debe tener una de estas raíces.
Si x 1 es raíz de x 2 5x k entonces 12 5 1 k 0 k 5 1 4 .
Si x 2 es raíz de x 2 5x k entonces 22 5 2 k 0 k 10 4 6 .
Por lo tanto, la suma de los posibles valores de k es 4 6 10 (E).
4.2.8
Probando mediante Ruffini encontramos dos soluciones reales (El Teorema de las Raíces
racionales reduce la búsqueda a 1 , 1 , 2 , 2 ):
1 2a 2a 2 4a 3 2
1 -1 2a 1 4a 1 2
1 2a 1 4a 1 2 0
2 2 4a 2 2
1 2a 1 1 0
Luego
x 4 2ax 3 2a 2x 2 4a 3x 2 ( x 1)( x 2) x 2 (2a 1) x 1
Así pues, este polinomio tendrá las cuatro raíces reales si y solo si
x 2 (2a 1) x 1
4.2.9
Sea x el primero de dichos números. Está claro que x 0 pues 0 1 2 3 80 1 2 .
x( x 1)( x 2) 8( x x 1 x 2) 24( x 1)
Puesto que x 0 podemos simplificar:
6
x( x 2) 24 x 2 2 x 24 0 x
4
Para x 4 42 52 62 16 25 36 77
4.2.10
Probando diferentes combinaciones llegamos a
x 4 2 x3 6 x 2 2 x 1 x 2 x 1 5 x 2
2
Y por tanto
0 x 4 2 x3 6 x 2 2 x 1 x 2 x 1 5 x x 2 x 1 5 x
x 2 x 1 5 x 0
x 2 x 1 5 x 0
Estas dos últimas ecuaciones son de segundo grado y se resuelven con los métodos
convencionales:
0 x x 1 5x x 1 5 x 1 x
2 2
1 5 1 5 4 1 (1)
2
0 x x 1 5x x 1 5 x 1 x
2 2
1 5 1 5 4 1 (1)
2
4.2.11
Realizamos la división sintética y vemos la pauta que sigue el cociente:
La respuesta correcta es (A)
4.2.12
Se puede llegar probando o mediante diferencia de cuadrados:
x 4
2
1 x8 2 x 4 1 x8 x 4 1 x 4
2
x8 x 4 1 x 4 1 x 4 x 4 1 x 2 x 4 1 x 2 x 4 x 2 1 x 4 x 2 1
4.3.1
(D) basta aplicar el Teorema del Residuo.
4.3.2
(D) basta aplicar el Teorema del Residuo.
4.3.3
Aplicando el Teorema del Residuo, sabemos que x 3 será una raíz de
p( x) 3x 3 9 x 2 k x 12 , luego
0 p(3) 3 33 9 32 k 3 12 34 34 k 3 12 0 3k 12 k 4
3 -9 4 -12
3 9 0 12
3 0 4 0
Es decir:
p( x) 3x 3 9 x 2 4 x 12 3x 2 4 x 3
Y la respuesta correcta es (C)
4.3.4
Podemos escribir
x100 x 2 3x 2 q( x) r ( x) para cierto polinomio r (x) de grado menor que 2.
Pero vemos que x 2 3x 2 x 2x 1
Por tanto:
x100 1x 2 3x 2 q( x) r ( x) x 2x 1 q( x) r ( x)
y en consecuencia:
1 1100 1 21 1 q(1) r (1) 0q( x) r (1) r (1) r (1) 1
2100 2 22 1 q(2) r (2) 0q(2) r (2) r (2) r (2) 2100
Entre las cinco opciones posibles, vemos que la única que satisface r (1) 1 , r (2) 2100 es (B).
Observación.
Se podría determinar r (x) . En efecto: r ( x) ax b , puesto que tiene grado menor que 2, y por
tanto:
1 r (1) a b a 2 1
100
2100 r (2) 2a b
b 1 a 1 2 1 1 2 1 2 2
100 100 100
r ( x) ax b 2 100
1 x 2 2
100
2
100
x x 2 2100 2100x 1 x 2 (B)
4.3.5
Aplicando el Teorema del Residuo tenemos que P(19) 99 y P(99) 19 , y estos datos ya son
suficientes para calcular el residuo de este polinomio al dividirlo entre x 19x 99 . Puesto
que estamos dividiendo entre un polinomio de segundo grado, el residuo será de primer grado:
P( x) x 19x 99Q( x) ax b
4.3.6
Si P( x) x1000 a x 2 9 Q( x)( x 1) , entonces, evaluando en x 1 tenemos:
0 Q(1)(1 1) (1)1000 a (1)2 9 1 a 9 a 10
4.3.7
Sean q( x) x 2 x 6 i p( x) 2 x4 x3 ax2 bx a b 1.
No podemos aplicar el Teorema del Residuo directamente porque el polinomio q(x) es de
segundo grado, y el Teorema del Residuo solo se aplica a ciertos polinomios de primer grado.
Pero observamos que q( x) x2 x 6 ( x 3)( x 2)
Y por tanto q(x) si es un factor de p(x) , por la transitividad de la divisibilidad, también lo
serán los polinomios x 3 i x 2 .
Y con ellos sí podemos aplicar el Teorema del residuo:
0 p(3) 2(3)4 (3)3 a(3)2 b(3) a b 1 162 27 9a 3b a b 1
8a 2b 134
0 p(2) 2 24 23 a 22 b 2 a b 1 3a 3b 39
Finalmente, solo hay que resolver el sistema
0 8a 2b 134 0 4a b 67 4a b 67 80
5a 80 a 16
0 3a 3b 39 0 a b 13 a b 13 5
b a 13 16 13 5
Y la solución es a 16 , b 5 .
4.4.1
Observamos que los cinco polinomios presentados cumplen p(1) 2018 0 y p(0) 1 0 .
Observamos también que todos los polinomios tienen potencias de x de grado impar, y que en
el polinomio B se encuentran los exponentes menores (excepto E)
Sabemos que si a 1,0 , entonces a n a m si n m y n, m son impares, luego las posibles
raíces de A, C y D siempre serán menores (más negativas) que las posibles raíces de B.
Así pues, solo nos queda comparar B y E.
2018
La raíz de E es a 0.99 .
2019
1
Observamos que 0.50 es aproximadamente una raíz del polinomio B:
2
1 1 1 2018 1 2018 1 1
17 11
2018 1 17 11 1 17 1 17 1 0
2 2 2 2 2 2048 2 2
4.4.2
Observando la gráfica vemos que a 0 , c 0 , y tiene dos raíces, una negativa p 0 y otra
positiva q 0 con q p p q 0 .
Luego b ( p q) 0 , y por tanto bc 0 . (D)
Observación: Un razonamiento tal vez más elegante sería observando la posición del vértice
b
x 0
2a
4.4.3
Observamos que se trata de una función de segundo grado, cuya gráfica es simétrica. Esta
función tiene puntos de corte con el eje X en -6 y 1:
Luego f (7) f (1 6) f (6 6) f (12) (B)
4.5.1
x3 3x 2 4 x 11 0 ( x a)( x b)( x c)
( x 2 (a b) x ab)( x c) x3 (a b c) x 2 (ab bc ac) x abc
3 (a b c) 3 a b c
Luego 4 ab bc ac
11 abc 11 abc
4.5.2
f (2) 2 2 2a b 2a b 4
f (4) 4 2 4a b 4a b 16
Observamos que en el caso f (2) f (4) el enunciado no nos está diciendo que el polinomio
g (x) tenga una única solución doble, solo que una de sus dos soluciones es f (2) f (4) .
En total hay 69 soluciones (recordemos que a 6 ), que junto a las 441 del primer caso hacen
un total de 510.
4.5.3
El enunciado nos está diciendo que P(3), P(4), P(a), P(b) son raíces del polinomio P(x) , que
solo puede tener dos raíces diferentes, digamos m, n .
Por Vieta sabemos que podemos escribir P( x) x2 (m n) x mn
Por lo tanto se pueden dar fundamentalmente dos casos:
a) m P(3) P(4), n P(a) P(b)
P(3) P(4) 32 3(m n) mn 42 4(m n) mn
9 3(m n) 16 4(m n) m n 7
Y por tanto
7 5
a m n 3 3 3
2 2 5 7
ab 6 (a b) 2 (6) 36
7 7 2 2
b m n 4 3 4
2
2
Finalmente: 49 36 85
4.5.4
P( x ) x 2 3 x 7
Q( x) x2 a x 2
R( x ) x 2 b x c
P( x) Q( x) 2 x2 (a 3) x 5
P( x) R( x) 2 x 2 (b 3) x (c 7)
Q( x) R( x) 2 x2 (a b) x (c 2)
a 2(t r ) 3
3
b 2(r s) 3 2(t r ) 3 2(r s) 3 2( s t ) 4r 6 0 r
2
a b 2( s t )
3
a 3 2 t 2
3 a 3 2t 3
5 2t 5 3t t 5 / 3
2 c 7 3s
3 b 3 3 2s c 52 / 19
b 3 2 s 5
2 c 7 3s c 2 2s 3 s 27 / 19
3 a b 2( s t )
c 7 2 s
2 c 2 2st
a b 2( s t )
c 2 2st
52
La solución es R(0) c .
19
4.5.5
Podemos escribir
p( x) x z1 x z2 ...x z673
3 3 3
x z1 x z1 x z1 x z2 x z2 x z2 ...x z673 x z673 x z673
Con lo cual tenemos las 3 673 2019 raíces de un polinomio de grado 2019, que se repiten en
grupos de tres:
z1 , z1 , z1 , z2 , z2 , z2 , ..., z673, z673, z673
Podemos aplicar las fórmulas de Vieta (y teniendo mucho cuidado en observar que los 2019
números complejos van en grupos de tres en tres):
20
20 z1 z1 z1 z2 z2 z2 ... z673 z673 z673 3 zi zi
1i 673 1i 673 3
1
19 3 zi2 9 zi z j 3 zi2 9 zi z j zi2 19 9 zi z j
i i j i i j i 3 i j
Sea S z j z k . Entonces zi2 19 9S
1
1 j k 673 i 3
Y por tanto
343
19 9S 2S S
400 1
9 3 9
343
Y como nos piden su valor absoluto, el resultado es .
9
4.5.6
b c
ax 2 bx c 0 x 2 x 0
a a
b) a c r, b s
Las condiciones (*) quedan ahora:
s r
(r s) , 1 r s (*)
r r
Si s 0 entonces:
0 r r 0 s absurdo.
1
Si s 0 entonces: r y por tanto
s
s 1
(r s) s (r 2 r s) s 2 1 s 3 1 s 2 s 3 s 2 1 0
r s
Trazando las gráficas de las funciones s 3 y 1 s 2 vemos que se cortan en un único punto,
que da lugar a una única solución, otro caso.
c) b c r , a s
Las condiciones (*) quedan ahora:
r r
( r s ) , r s
s s
Si r 0 tenemos
0
0 (0 s) s 0 r , absurdo.
s
Si r 0 tenemos
r 1
r s s s2 1
s s
Estamos ante un caso similar al caso a), luego generará otro caso.
4.5.7
9 32 x y z x 2 y 2 z 2 2( xy yz xz )
2
9 3 2xy yz xz
6 2xy yz xz
3 xy yz xz
x3 y 3 z 3 3xyz ( x y z ) x 2 y 2 z 2 ( xy yz xz )
3 3xyz 33 3 0 3 3xyz xyz 1
Entonces, aplicando fórmulas de Vieta, y aplicando las igualdades que hemos obtenido en la
versión anterior:
a x yz 3
b xy yz xz 3
c xyz 1
Así pues, el polinomio es 0 t 3 3t 2 3t 1 t 1 , cuyas única solución es t 1 (triple).
3
Así pues, x y z 1
x a ,
i 1
k
i
i 1
k
i k 1,2,...n
Donde las ai son constantes, cuyas únicas soluciones son x1, x2 ,..., xn a1, a2 ,..., an y todas
sus posibles permutaciones.
(Ver USA Mathematical Olympiads 1972-1986, Murray S. Klamkin, página 18)
4.5.8
Sean a y b las soluciones de la ecuación.
13 15 a b 13 / 2
2 x 2 13x 15 0 x 2 x 0
2 2 ab 15 / 2
13 13 3
Si, además, suponemos a 5 , aislando de la primera ecuación 5 b b 5
2 2 2
Observación: También podríamos haber aislado de la segunda ecuación:
15 3
5b 15 / 2 b 5
2 2
4.5.9
4.5.10
Sean a y b las soluciones de la ecuación.
4 1 a b 4 / 3
3x 2 4 x 1 0 x 2 x 0
3 3 ab 1 / 3
1 1 b a a b 4/3
4
a b ab ab ab 1/ 3
4.5.11
Sean a y b las soluciones de la ecuación.
11 5 a b 11 / 2
2 x 2 11x 5 0 x 2
x 0
2 2 ab 5 / 2
1 1 b a a b 11 / 2 11
a b ab ab ab 5/ 2 5
4.5.12
Sean a y b las soluciones de la ecuación.
5 3 a b 5 / 2
2 x 2 5x 3 0 x 2 x 0
2 2 ab 3 / 2
3 25 25 37
(a b) 2 a 2 b 2 2ab (5 / 2) 2 a 2 b 2 2 a2 b2 3 a2 b2 3
2 4 4 4
4.5.13
Sean a y b las soluciones de la ecuación.
2 x 2 3x 5 0
3 5 a b 3 / 2
2 x 2 3x 5 0 x 2 x 0
2 2 ab 5 / 2
5 3
(a 1)(b 1) ab a b 1 ab (a b) 1 1 0 (B)
2 2
4.5.14
Aplicando la fórmulas de Vieta, sabemos que m n 2018 , m n 1 . Luego
m 1 n 2018 m n (1 n)n n n 2 2018 n 2 n
1 m n
2019 2018 1 n 2 n m n n 2 m
4.5.15
Aplicando las fórmulas de Vieta:
Sean a y b las raíces de la ecuación x 2 k x k 0 . Sabemos que:
a b k
ab k
Luego
a b 2 a 2 b2 2ab a 2 b2 a b 2 2ab k 2 2k 3
k 3
k 2 2k 3 0
k 1
y por tanto, el mayor valor del parámetro k es 3 (E).
4.5.16
p 2 2 p q 2 2q 15 2 p q
p 2 2 p q 2 2q 2 pq 15 0
p 2 q 2 2 pq 15 2( p q) 15 0
p q 2 2( p q) 15 0
Sea k p q , luego
k 5
k 2 2k 15 0
k 3
Puesto que ambos son positivos, el único resultado aceptable es p q k 5 (B)
4.5.17
ab 1
i) Aplicando las Fórmulas de Vieta sabemos que , luego
a b 3
P a 2b 2 ab (1) 2 1
2
4.5.18
39 (ver 4.5.12)
4.5.19
15 / 16 (ver 4.5.11)
4.5.20
44 (ver 4.5.13)
4.5.21
Sean a y b las raíces de dicha ecuación.
2003 1 2003 2 2004 2004
x 1 0 x x 1 0 x2 x 0
2004 x 2004 2003 2003
y por tanto:
1 1 a b 2004 / 2003
1
a b ab 2004 / 2003
4.5.22
Sean a y b las raíces de la ecuación 2 x 2 m x 8 0 .
m
2x2 m x 8 0 x2 x 4 0 , y aplicando las fórmulas de Vieta tenemos:
2
ab 4
a b m / 2
Además, suponemos a b m 1.
Luego:
a b m / 2 m m 2m 2 3m 2 3m 2
2a m 1 a
a b m 1 2 2 2 4
3m 2 3m 2 4m 4 m 2
b a m 1 m 1
4 4 4
3m 2 m 2
4 ab 43 (3m 2)(m 2) , y esta última ecuación de
4 4
segundo grado tiene como soluciones 6 y 10 / 3 .
4.5.23
Aplicando las fórmulas de Vieta en ambas ecuaciones sabemos que ab 2 y que
1 1 1 1 11 1 1 9
q a b ab a b 2 11 4 (D)
b a a b ab 2 2 2
4.5.24
Sean a, b, c, d , e, f las raíces de este polinomio. Aplicando las fórmulas de Vieta, sabemos que
a b c d e f 16 y a b c d e f 10
Nos piden el coeficiente B, que se determina (con signo menos) con todos los productos
ordenados de 3 elementos:
abc+abd+abe+abf+acd+ace+acf+ade+adf+aef+
+ b c d + b c e + b c f + b d e + b d f + b e f + c d e + c d f + c e f + d e f = 88
Observación: Una forma tal vez más elegante de llegar a este resultado evitanto esta suma tan
tediosa es la siguiente:
De a b c d 2 , e f 1 se deduce que el polinomio es
x 12 x 24 x 2 2x 1x 4 8x3 24x 2 32 x 16
Y calculando únicamente los términos que dan lugar a x 3 llegamos a B 32 48 8 88 .
4.5.25
Aplicando las fórmulas de Vieta:
ab 3
a b 23
Luego
232 (a b)2 a 2 b2 2ab a 2 b2 2 3 a 2 b2 232 6 523
4.6.1
Sea p(x) un polinomio de tercer grado con soluciones a, b, c .
Aplicando las Identidades de Newton tenemos que
P0 a 0 b 0 c 0 3
P1 a1 b1 c1 A 1
P2 a 2 b 2 c 2 AP1 2 B 2 2 1 2 B B 1 / 2
1 1
P3 a 3 b 3 c 3 AP2 BP1 CP0 3 3 1 2 1 C 3 C
2 6
Así pues, C abc 1 / 6
4.6.2
1 12 x y z x 2 y 2 z 2 2xy yz xz 35 2xy yz xz
2
xy yz xz 17
1 13 x y z x 3 y 3 z 3 3xyz 3xy yz xz x y z
3
Y aplicando las Identidades de Newton, tenemos que x, y, z serán las raíces del polinomio
pw w3 e1w2 e2 w e3 w 1w 3w 5
Nota: Una solución del sistema se podría haber encontrado determinando las soluciones enteras
de la ecuación xyz 15 , con las técnicas básicas de Teoría de Números, pues
x, y, z 1 , 3 , 5 , 15, para después deducir que todas las soluciones posibles son estas
aplicando las Identidades de Newton anteriores.
4.6.3
Este ejercicio se resuelve fácilmente mediante las Identidades de Newton introducidas en el
Tema 14 de este documento. Aquí exponemos una solución alternativa:
Sean p, q, r las tres raíces del polinomio. Entonces se cumple
p 3 5 p 2 8 p 13 0
q 3 5q 2 8q 13 0
r 3 5r 2 8r 13 0
Y por tanto
p3 q3 r 3 5 p2 q2 r 2 8 p q r 39 0
Es decir:
s3 5s2 8s1 39 0 s3 5s2 8s1 13s0
4.6.4
Pz1 Pz2 Pz3 Pz4 x16 x15 x13 x12 x11 x26 x25 x23 x22 x12
x36 x35 x33 x32 x31 x46 x45 x43 x42 x14
x16 x26 x36 x46 x15 x25 x35 x45 x13 x23 x33 x43 x12 x22 x32 x42
x11 x12 x31 x14 P6 P5 P3 P2 P1
Luego todo se reduce a determinar los Pk x1k x2k x3k x4k siguiendo la notación de las
Igualdades de Newton:
Q( x) x 4 x3 x 2 1 x 4 x3 x 2 0 x 1.
P1 (1) 0 P1 1
P2 P1 2 (1) 0 P2 1 2 0 P2 3
P3 P2 P1 3 0 0 P3 3 1 0 0 P3 4
P4 P3 P2 0 P1 4(1) 0 P4 4 3 4 P4 11
P5 P4 P3 0 P2 1 P1 0 P5 11 4 0 1 0 P5 16
P6 P5 P4 0 P3 1 P2 0 P6 16 11 0 4 3 0 P6 30
5.1
Primera versión.
ax b
a b
cx d aax b bcx d a 2 x ab bcx bd
f f ( x)
ax b cax b d cx d acx bc cdx d 2
c d
cx d
a 2 bc x ba d
x
ca d x bc d 2
px q
Para que se cumpla x se tiene que cumplir r q 0 y p s 0
rx s
En nuestro caso:
ca d ba d 0
0 a 2 bc bc d 2 a 2 d 2
Puesto que b, c 0 , de la primera igualdad tenemos que a d . Así pues, nuestra función
queda de la forma:
ax b
f ( x)
cx a
Esta función no está definida en x a / c , y este valor lo podemos determinar con las otras dos
condiciones del enunciado:
19a b
19 f (19) 1919c a 19a b 192 c 19a 19a b 192 c 38a b
19c a
97a b
97 f (97) 9797c a 97a b 97 2 c 97a 97a b 97 2 c 194a b
97c a
97 c 194a b b 97 c 194a
2 2
a
156a 9048c a 58c 58
c
La solución es 58.
Segunda versión.
Podríamos haber llegado a a d teniendo en cuenta que
b
a b
b
f (0) f f f (0) 0 d
b b
0 0 a b ab bd b(a d )
d d b
c d d
d
a d
Fuente de estas versiones: [Link]
5.2
n 2019 n
Vemos que f f
2019 2019
n n 2019 n
2 2 2 2
n n
f 1
2019 2019 2019 2019 2019
2019 n 2019 n 2019 n 2019 n n
2 2 2 2
f 1
2019 2019 2019 2019 2019
Vemos que los términos se cancelan por parejas, y al haber 1009 parejas en esta suma, el
resultado final es cero (A).
5.3
3x 6 12 x 3 y 3x 3 x 3 4 y
3x x3 4 y
3x x3 4 y
x8 16 x 2 y 2 x 2 x 6 16 y 2 x 6 16 y 2 2
x 3 4 y
2
3x x3 4 y
3
3x
x 4y x 4y
3 3
x 4y
6.1
1 1 1 1 1 1
a b c 1 1 1
b c a a b c
ab 1 bc 1 ac 1 a 1 b 1 c 1
b c a a b c
ab 1bc 1ac 1 a 1b 1c 1
abc abc
ab 1bc 1ac 1 a 1b 1c 1
a 2b 2c 2 a 2bc b 2 ac c 2bc ac ab bc 1 abc ac bc c ab a b 1
132 13(a b c) 13 a b c
Definiendo s a b c , la ecuación 132 13s 13 s tiene como solución s 13 , y por
tanto a b c 13 .
6.2
De la igualdad del enunciado deducimos que
bc ca a b
ab , bc , ca
bc ca ab
Multiplicando las tres igualdades anteriores llegamos a abc 1, de donde se deduce que
2
abc 1 .
Fuente de la solución: 101 Problems in Algebra from the training of the USA IMO team (Adreescu, Feng, 2001), pág. 2
6.3
440 n! n 1!n 2! n!(n 1) n!(n 1)(n 2) n!(n 1)(1 n 2) n!(n 1)(n 3)
440 n! n!(n 1)(n 3) 440 (n 1)(n 3)
(n 1)(n 3) 440 23 5 11
Probando posibles combinaciones (o resolviendo la ecuación cuadrática resultante) vemos que
una que satisface esta condición es
n 1 2 2 5 20
n 19
n 3 2 11 22
Y por tanto la respuesta correcta es 1 9 10 (C ) .
6.4
Sumando la primera y la segunda ecuación:
ab c 2d bc a 8
Así pues, b d 3 b 3 d
Llegando finalmente a:
d 2c cd 3
c 0a 20 2
cd c d 3
Y por la cuarta ecuación del sistema original:
6 da b c 2d b
3 2(6 2d ) d 12 4d d 12 3d 9 3d d 3
3 ab c d 2b d
b 6 23 0
Quedaría comprobar el caso b d . Pero en este caso la primera y la cuarta ecuación forman el
sistema
ab c b 3
ab b c 6
6.5
a b c d 2c d c d 2cd
2
Y el único número de la lista que no se puede representar de esta forma es 100 (B)
6.6
17 x 51y 102 17 x 3 17 y 6 17 x 3 y 6 9 x 27 y 6 9 54 (A)
6.7
Está claro que I no es siempre cierto.
2 2 2
m 0 0 n m m m n , luego II es siempre cierta.
m m m
2 m 2 2
nm nm m2 2 m2 , luego III es siempre cierta.
m m
Así pues, la respuesta correcta es D.
6.8
Sumando y restando las dos expresiones:
x y z z r 2x
2 x (2) y ( z r )( z r ) z 2 r 2
x y r z r 2 y
z2 r2 r2 z2
xy
4 4
y la solución es (D).
6.9
Sumando las tres ecuaciones:
ab c
b c d a 2b 2c d c d a 2b c 0 c 2b
c d a
a b c a b 2b a 3b
b c d b 2b d d b
c d a 2b b a 3b a
Luego, finalmente, a b c d 3b b 2b b 5b 5 pues b 1.
7.1.1
Elevamos al cuadrado la segunda ecuación:
2
x3
x3
2 x6
1010 1010 2 1020
y y y
xy 2 108
x3 x3
1010 x3 1010 y y 10
104
1012
3
102 100
y 10 1010 1010
Por lo tanto, la única solución es x 104 10000 , y 100 .
7.1.2
Denotamos los 15 números por a1 10, a2 , a3 , ..., a15 . Sea S la suma de siete números
consecutivos.
Nota: De hecho, con este razonamiento vemos que la única solución posible es
a1 a2 a3 ... a15 10 .
7.1.3
Aislamos la x por el método convencional, mediante operaciones contrarias sucesivas:
144 144 2 53 38 53
2
53 53 53 38
53 53 38 15 38
1
38 38 38 15
38 38 2 15 8
2
15 15 15
2 8 8 8 15 15 12 3
4(3 x) 15 x 3 ( A)
3 x 15 4(3 x) 15 4 4 4
7.1.4
Si A es invertible, A3 A se simplifica en A2 I , es decir:
1 / 4 a / 2 1
1 / 4 b / 2 0
1 / 2 1 / 2 1 / 2 1 / 2 1 0 a 3 / 2
a b a b 0 1 a / 2 ab 0 b 1 / 2
a / 2 b 1
2
1 / 2 1 / 2 1 / 2 1 / 2 1 / 2 1 / 2 1 / 4 a / 2 1 / 2 a / 2
A A2
a a a a a a a / 2 a 2 a / 2 a 2
1 / 2 1/ 2 1 / 4 a / 2 1 / 2 a / 2 (1 / 4 a a 2 ) / 2 (1/ 4 a a 2 ) / 2
A3
a a a / 2 a 2 a / 2 a 2 a(1/ 4 a a 2 ) a(1/ 4 a a 2 )
(1 / 4 a a 2 ) / 2 (1 / 4 a a 2 ) / 2 1 / 2 1 / 2
A A
3
a (1 / 4 a a 2
) a (1 / 4 a a 2
) a a
1
a a 1
2
4
a(1 / 4 a a 2 ) a
7.1.5
Mediante el cambio de variable x 2a , y 2b , z 2c obtenemos las ecuaciones
x y z 7
1 1 3
x y 4
Y una tercera ecuación:
x y z 2a 2b 2c 2abc 23 8 (*)
1 1 3 x y 3 7z 3 z (7 z ) 3
x y 4 xy 4 8/ z 4 8 4
z (7 z )
3 z (7 z ) 6 7 z z 2 6 0 z 2 7 z 6 ( z 1)( z 6)
2
Si z 1 , llegamos al sistema
x y 6
1 1 3 1 3 1 3y 4 4y
x
x y 4 x 4 y 4y 3y 4
xy 8
4y
y 6 4 y y (3 y 4) 6(3 y 4)
3y 4
4 y 3 y 2 4 y 18 y 24
3 y 2 18 y 24 0
y 2
y 2 6 y 8 0 ( y 2)( y 4) 0
y 4
y 2 2b b 1 , x 4 2 a a 2 , c 0
y 4 2b b 2 , x 2 2 a a 1 , c 0
Si z 6 , llegamos al sistema
x y 1
1 3 1 3y 4 4y 4y
1 1 3 x 1 y
x y 4 x 4 y 4 y 3 y 4 3 y 4
(1 y )(3 y 4) 4 y 3 y 2 7 y 4 4 y 3 y 2 3 y 4 0
Y esta última ecuación no tiene solucion real.
Observación.
Sin haber deducido la tercera ecuación (*) también se podría haber resuelto el problema, pero
llegando a una ecuación de tercer grado muy incómoda:
23
a b c 3 c 3 a b 7 2 a 2b 23ab 2 a 2b a b
2 2
1 1 3
2 a 2 b 3 / 4 a b
2 2 4
3a b 23
a b c 3 c 3a b 7 2 2 2 a b
2 2 a b
a b
2 2
1 1 3
2 a 2 b 3 / 4 a
b
2 2 4
8 4y 8 3y 4
7 x y 7 y
xy 3y 4 y 4 y
4 y y (3 y 4) 6 y 8
7
3y 4 y2
3y2 6y 8
7
3y 4 y2
3 y 4 (6 y 8)(3 y 4)
7
(3 y 4) y 2
7(3 y 4) y 2 3 y 4 (6 y 8)(3 y 4)
3 y 4 21 y 3 46 y 2 48 y 32 0
7.2.1
2
a) x 2 2 2 x 2 2 15 0
Con el cambio de variable a x 2 2 convertimos la ecuación en una ecuación de segundo
grado que se resuelve mediante los métodos convencionales:
a 5
a 2 2a 15 0 (a 5)(a 3) 0
a 3
Ahora deshacemos el cambio de variable:
a 5 5 x2 2 x2 3 x 3
a 3 3 x2 2 x 2 5 y esta ecuación no tiene soluciones reales.
Las soluciones son x 3, x 3
b) De la misma forma, con un cambio de variable: y 3, y 3 .
c) De la misma forma, con un cambio de variable: y 1, y 2 .
7.2.2
Haciendo el cambio de variable a r 2 5r , la ecuación queda:
a 4
aa 3 4 a 2 3a 4 0 (a 4)(a 1) 0
a 1
Deshaciendo ahora el cambio de variable:
r 1
a 4 4 r 2 5r r 2 5r 4 0 r 1r 4 0
r 4
5 52 4 1 (1) 5 25 4
a 1 1 r 5r r 5r 1 0 r
2 2
2 2
5 29
2
7.2.3
Observamos que
xx 3 x 2 3x
x 1x 2 x2 2 x x 2 x2 3x 2
Por lo tanto, mediante el cambio de variable a x2 3x 1, la ecuación del enunciado queda:
(a 1)(a 1) 1 2712
a 2 12 1 2712
a 2 2712 a 271
x 18
a 271 271 x 2 3x 1 x 2 3x 270 0
x 15
a 271 271 x2 3x 1 x2 3x 272 0
Puesto que la segunda ecuación no tiene soluciones reales, las únicas soluciones son
x 18 , x 15
7.2.4
Mediante un cambio de variable y x 2 x 1 , la ecuación se convierte en
y 3
y y 1 12 y 2 y 12 0 ( y 4)( y 3) 0
y 4
Y ahora deshacemos el cambio de variable:
x 2
y x 2 x 1 3 x 2 x 2 0 ( x 2)( x 1) 0
x 1
y x 2 x 1 4 x 2 x 5 0
Y esta segunda ecuación no tiene soluciones reales, pues su discriminante es menor que cero:
(1)2 4 1 5 1 20 19 0
7.2.5
9 f (k ) k 2 7k k k 2 6k k 2 6k 9 0 k 3
f (1) (1)2 7(1) 3 1 7 3 11 (E)
7.2.6
Con el cambio de variable u r 2 5r , la ecuación se transforma en
u 4
u u 3 4 u 2 3u 4 0
u 1
Deshaciendo el cambio de variable:
r 4
r 2 5r 4 r 2 5r 4 0
r 1
5 29
r 2 5r 1 r 2 5r 1 0 r
2
7.3.1
Factorizamos las ecuaciones:
336 3x 2 2 xy x3x 2 y
84 2 y 2 3xy y2 y 3x y 3x 2 y
7.3.2
16 r 2 2rs r (r 2s)
4 rs 2s 2 s(r 2s)
7.3.3
Observamos que
40 x2 y 2 x2 y 2 2 xy xy x y
2 2
i por tanto, con el cambio de variable a xy , b x y , el sistema queda
a 2 b 2 40
a b 8
b 6 x y
y 3 7 x 6 3 7 3 7
y 3 7 x 6 3 7 3 7
a 6 xy
2 y x 6 (2 y ) y 2 y y y 2 y 6 0
2 2
b 2 x y
Esta última ecuación no tiene soluciones reales, y por tanto no hay ninguna solución más. Las
únicas soluciones son: x 3 7 , y x 3 7 i x 3 7 , y 3 7 .
7.3.4
x y 2 x 2 y 2 2 xy 25 2 12 25 24 49 x y
7
7
x y 7
y 7 x x(7 x) 12 x 7 x 12
2
xy 12
x 4 y 7 4 3
x 2 7 x 12 0
x 3 y 7 3 4
x y 7
y 7 x x(7 x) 12 x 7 x 12 x 7 x 12 0
2 2
xy 12
x 4 y 7 (4) 3
x 2 7 x 12 0
x 3 y 7 (3) 4
Luego hay cuatro soluciones: ( x, y) (4,3) , (3,4) , (4,3) , (3,4)
7.4.1
Hacemos el cambio de variable a 4 x 3 , y la ecuación del enunciado queda:
10 a 2 10 a 2 10
a 7 7 7 a 2 10 7a a 2 7a 10 0
a a a a
a 2
(a 2)(a 5) 0
a 5
Y ahora deshacemos el cambio de variable:
7
a 2 2 4 x 3 22 4 x 3 4 3 4 x 7 4 x x
4
a 5 5 4 x 3 5 4 x 3 25 3 4 x 28 4 x x 7
2
7.4.2
Mediante el cambio de variable y x 2 9 x 30 la ecuación del enunciado se convierte en
y 2 y 15 y 2 4( y 15) y 2 4 y 60 y 2 4 y 60 0
y 10 0 y 10
y 2 4 y 60 0 ( y 10)( y 6) 0
y 6 0 y 6
Sin embargo, una raíz es siempre positiva, por lo tanto la opción y 6 no es aceptable, pues
en ese caso tendríamos
6 2 6 15 2 9 6
x 4
y x 2 9 x 30 10 x 2 9 x 20 0 ( x 4)( x 5) 0
x 5
7.4.3
3
25 2 3 1 x 3 25 2 3 1 x 27 2 3 1 x 2
2 3 1 x 4 3 1 x 2 1 x 8 x 7
7.4.4
Elevamos al cuadrado la ecuación:
x 2
2
18 x 30 4 x 2 18 x 45 4 x 2 18 x 30 15
4 x 2 18 x 30 4 15 4 x 2 18 x 30 60
Mediante la substitución y x 2 18x 30 , la ecuación anterior se convierte en
y 6
y 2 4 y 60 y 2 4 y 60 0
y 10
La solución y 6 , aplicada a la ecuación del enunciado genera una parte izquierda negativa,
lo que nos lleva al absurdo, pues ninguna raíz cuadrada puede ser negativa. Estos dos valores
no se pueden tener en cuenta.
La solución y 10 implica x 2 18x 30 10 x 2 18x 20 0 , cuyas soluciones son
x 9 61 , y cuyo producto es
9
61 9 61 (9) 2 61
2
81 61 20
7.5.1
Haciendo el cambio de variable a x , b y , el sistema queda
a b 7
3a 4b 14
7.5.2
Haciendo el cambio de variable a x y a 2 x y
La primera ecuación queda:
a 6
a 2 a 30 a 2 a 30 0 0 (a 6)(a 5)
a 5
Pero como estamos suponiendo que a es una raíz cuadrada, no puede ser negativa, y por tanto
solo queda que
a 5 a 2 25 x y
8.1
m b 0 significa que m y b son diferentes de cero y con el mismo signo.
El valor de m es la pendiente de la recta, y el valor de b es el punto de corte con el eje Y.
Si ambos son positivos, no se pueden dar los casos B, D y E.
Si ambos son negativos, no se pueden dar los casos A, C y E.
En todo caso no se puede dar el caso (E).
8.2
Vemos que esta función lineal es, por un lado, creciente, y por otro decreciente, luego solo
puede ser constante. Por tanto, f (0) f (5) 5 (D).
8.3
f ( x) a x b
12 f (6) f (2) a 6 b a 2 b 6a 2a 4a a 3
f (12) f (2) a 12 b a 2 b a 12 b a 2 b 12a 2a 10a 10 3 30 (D)
8.4
Aplicando la definición de función inversa,
x f 1 f x bax b a abx b2 a
Tomando, en particular, x 0 b2 a 0 , x 1 ab b2 a 1
Luego ab 1
Multiplicando la primera ecuación por b,
b3 ab 0 b 0 b3 1 b 1 a 1 , y por tanto a b 2 (A).
8.5
G 2 9(C 2)
7C 2 9(C 2) 9C 18 18 2 9(C 2) 9C 7C
G 7C
16
16 2C C 8 G 7 8 56
2
G x 5(C x) 56 x 5(8 x) 40 5 x 56 40 5 x x
16
16 4 x x 4
4
Tienen que pasar cuatro años.
8.6
Sumando las dos ecuaciones obtenemos la ecuación
10000 x 10000 y 50000
Que la podemos simplificar llegando a
x y 5
de la misma forma, restando las dos ecuaciones obtenemosç
3502 x 3502 y 3502
Que se simplifica en
x y 1
Y el sistema resultante tiene solución x 3 , y 2 .
8.7
Llamando x, y, z a los radios respectivos:
Llegamos al sistema lineal
x y 3 x 1
x z 4 y 2 A 1 2 3 14 (E)
2 2 2
y z 5 z 3
8.8
c p 34
c 24, p 14 (B)
c p 14
8.9
x = número de chicos, y = número de chicas.
40( x y)
y 40 x 40 y 100 y 2 x 3 y
100
30( x 2 y 2)
y 2 30 x 30 y 100 y 200 3x 7 y 20
100
Resolviendo el sistema resultante llegamos a la solución x 12 , y 8 (C)
8.10
2 x 7 3 x 2
Luego la segunda ecuación se debe satisfacer para x 2 :
b(2) 10 2 b 5 1 b 4 (B)
8.11
x= días en los que hace su trabajo bien
y = días en los que hace su trabajo excepcionalmente bien
x y 10 x 7
(A)
3x 5 y 36 y 3
8.12
x 2 y 361 x 121
y 2 x 362 y 120
8.13
Resolvemos el sistema
x y 10 x 7
(C)
5 x 2 y 29 y 2
8.14
Sumamos las dos ecuaciones:
3a 7b 1977
8a 8b 3984 a b 498
5a b 2007
8.15
x = número de canicas rojas.
y = número de canicas azules.
x 1 y
x 1 x 1 y 7( x 1) 7 x 7 y 6 x 6
7
x y2
x x y 2 5x y 2 4 x
5
El sistema resultante tiene por solución x 4 , y 18 , y el total de canicas es 22 (B)
8.16
14 abc bcd cda dab abc d bc ca ab abc 4d
14d abcd 4d 2 30 4d 2
4d 2 14d 30 0
2d 2 7d 15 0 d 5 , 3 / 2
Si d 5 :
14 abc 4(5) 14 abc 20 abc 6
Y el sistema queda:
a b 3
ab bc ca 4
abc 6
4 ab bc ca ab c(a b) ab c(3) ab 3c ab 3c 4
abc 6
Luego (3c 4)c 6 3c 2 4c 6 0 , es una ecuación de segundo grado con discriminante
16 4 3 6 0 , luego sus soluciones no son reales, no son aceptables.
Si d 3 / 2
14 abc 4(3 / 2) 14 abc 6 abc 20
Y el sistema queda:
a b 3
ab bc ca 4
abc 20
4 ab c(a b) ab c(3) ab 3c ab 3c 4
(3c 4)c 20
abc 20
4 (4) 2 4 3 (20) 4 256 4 16 10 / 3
3c 4c 20 0 c
2
23 23 2 3 2
Si c 10 / 3 , entonces: ab 3(10 / 3) 4 6
y por tanto:
9 (3)2 (a b) 2 a 2 b2 2ab a 2 b2 12 a 2 b2 9 12 3 0 absurdo.
9.1
Resolvemos la inecuación mediante la determinación de los puntos frontera:
x 2 a 1x 5a 2 a 2 1 x2 a 1x 5a 2 a 2 1 0
x 2 a 1x 5a 2 a 2 1 0
x
(a 1) (a 1) 2 4 1 5a 2(a 2 1 (a 1) 9(a 1) 2 (a 1) 3 a 1
2 2 2
9.2
Las negaciones de las tres frases del enunciado nos dan los intervalos 0,6 , 5, y 4, ,
cuya intersección es el intervalo 5,6 (D)
9.3
En primer lugar vemos que la base de la potencia es siempre positiva. En efecto, la función
f ( x) x 2 x 17 tiene discriminante (1)2 4 1 1 3 0 y la parábola pasa por
f (0) 3 0 .
Aplicamos logaritmos:
x 2
x 1 1 log x 2 x 1 log 1 0 x log x 2 x 1 0
x x
x 0 , log x 2 x 1 0 x 0 , x 2 x 1 1 x 0 , x 2 x 0 (1)
x 0 , log x 2 x 1 0 x 0 , x 2 x 1 1 x 0 , x 2 x 0 (2)
El primer sistema de desigualdades:
x 0
x 1
0 x x x( x 1) x 1 o x 0
2
9.4
Puesto que x es positivo,
x 2x x 2x
2 2
x 4x2 1 4x
1
4
x (A)
9.5
(E)
10.1
Si x 0, y 0
x x y 10 x x y 10 2 x y 10
2 12 y 10 y 14
x y y 12 x y y 12 x 12
y la solución no es aceptable pues contradice la hipótesis y 0 .
Si x 0, y 0
x x y 10 x x y 10
y 10
x y y 12 x y y 12
y la solución no es aceptable pues contradice la hipótesis y 0 .
Si x 0, y 0
x x y 10 x x y 10
y 10
x y y 12 x y y 12
y la solución no es aceptable pues contradice la hipótesis y 0 .
Si x 0, y 0
x x y 10 x x y 10 2 x y 10 14 32
y x
x y y 12
x y y 12 x 2 y 12 5 5
Y en este caso la solución sí es aceptable.
32 14 18
Luego x y
5 5 5
10.2
x p x p
p x 15
x 15 15 x
p x 15 0 x p 15 p 15 x p 15 p
x p 15 x p 15 15 p x
Luego , si p x 15 ,
f ( x) x p x 15 x p 15 x p 15 x 15 p x 30 x
10.3
Primera versión.
Puesto que las funciones de la derecha son todas lineales, podemos suponer que los polinomios
involucrados son todos de primer grado.
b
Una función del tipo a x b tiene un "extremo puntiagudo" cuando ax b 0 x
a
2 r (0) a 0 a c0 d 0 e a e 2 e 2 a
1 r (1) a (1) a c(1) d (1) 2 a c d 2 a d 3 a c
Si x 1 x 1 0 x 1 ( x 1) , y por tanto:
ax a a x 1 a ( x 1)
cx c x c x
Luego
1 r ( x) a ( x 1) c x 3 a c x 2 a
a x a c x 3 a c x 2 a
a c 3 a c 0
a c 3 a c x a 2 a
a 2 a 1
3
a 2 a 1 2 a 2 1 2 a 3 a
2
0 a c 3 a c a c 3 a c 0 2 a 3 a
3
2
Para encontrar el valor de c utilizamos r (1) 0
5 5
0 r (1) a a c d e 2 a c 3 a c 2 a 2 c 5 c
2 2
3 5 3 5 3 1
Tomando a y c , d 3 1 y e 2 comprobamos que satisface las
2 2 2 2 2 2
condiciones exigidas:
1 si x 1
3 3 5 1
r ( x) x x x 3x 2 si 1 x 0
2 2 2 2
2 x 2 si x 0
Segunda versión.
En la solución "del libro" a este problema se sigue el siguiente razonamiento:
Puesto que x 1 y x 0 son los dos puntos críticos de las funciones con valor absoluto,
podemos suponer que
(c a b) x d a si x 1
r ( x) a x 1 b x cx d (a c b) x a d si 1 x 0
(a b c) x a d si x 0
10.4
y 1 x
x y 1 y 1 x
y 1 x x 1
x y 1
y 1 x
x y 1 y 1 x
y 1 x x 1
Si, además, representamos x 3 y 3 , vemos que el conjunto de rectas se corta en tres puntos:
La solución es 3 (C)
10.5
a 1 a 0
Teniendo en cuenta que haciendo una tabla de valores:
a 1 a 0
a b c abc a b c a b c
a b c abc a b c a b c
a b c a b c
a b c (a b)
a b c a b c
>0 >0 <0 1 1 (1) (1) 0
<0 <0 >0 -1-1+1+1=0
>0 <0 a b c0 1-1-1+1=0
1-1+1-1=0
a b c0
10.6
a 1 a 0
Vemos que solo importa el signo de los números, pues , y por tanto podemos
a 1 a 0
suponer que todos los valores son 1 .
a b c abc
a b c
a b c abc
1 1 1 1+1+1+1=4
1 1 -1 1+1-1-1=0
1 -1 1 1-1+1-1=0
1 -1 -1 1-1-1+1=0
-1 1 1 -1+1+1-1=0
-1 1 -1 -1+1-1+1=0
-1 -1 1 -1-1+1+1=0
-1 -1 -1 -1-1-1-1=-4
11.1
5
Si x es un entero, x x , y por tanto la ecuación queda 2 x 5 x , lo cual es
2
absurdo pues x es un número entero.
Si x no es un entero, x x 1 , y por lo tanto resolvemos el sistema
x x 5 4
x x 1 5 2 x 4 x 2 2 x 3
x x 1 2
Luego el conjunto solución es 2 x 3 .
11.2
x 12 12 x 13 122 x 132 144 x 169
1202 1212
100 x 120 120 100 x 121 1202 100 x 1212
100
x
100
2 2
120 121
x 12 x 12.1
2 2
10 10
11.3
Sea f ( x) 2 x 4 x 6 x 8x . Nos piden determinar la imagen de esta función. La
función es claramente creciente.
Si x n un número entero, f (n) 2n 4n 6n 8n 20n
Teniendo en cuenta las gráficas anteriores, podemos determinar la imagen de f (x) para
0 x 1 . Para ello es muy útil dibujarlas todas superpuestas:
0 x 1/ 8 f ( x) 0 0 0 0 0 1/ 8 x 1/ 6 f ( x) 1 0 0 0 1
1/ 6 x 1/ 4 f ( x) 1 1 0 0 2 1/ 4 x 1/ 3 f ( x) 2 1 1 0 4
1/ 3 x 3 / 8 f ( x) 2 2 1 0 5 3 / 8 x 1/ 2 f ( x) 3 2 1 0 6
1/ 2 x 5 / 8 f ( x) 4 3 2 1 10 5 / 8 x 2 / 3 f ( x) 5 3 2 1 11
2 / 3 x 3 / 4 f ( x) 5 4 2 1 12 3 / 4 x 5 / 6 f ( x) 6 4 3 1 14
5 / 6 x 7 / 8 f ( x) 6 5 3 1 15 7 / 8 x 1 f ( x) 7 5 3 1 16
f (1) 20
Luego la función recorre todos los números entre el 0 y el 20 excepto 3, 7, 8, 9, 13, 17, 18 y 19.
Es decir, recorre 12 números de cada 20, y puesto que 1000 50 20 , recorrerá 50 12 600
números del total de 1000.
11.4
Representar la gráfica de la función f ( x) x no ofrece ningún problema, ni tampoco
representar la función f ( x) x 2 x 2 superponiéndola a la de la parábola
x 2 x 2 ( x 2)( x 1)
1 5 1 5
x 2 x 2 1 x 2 x 1 0 x y el intervalo es 1,
2 2
Y el segundo se encuentra en 2 x 3
1 17
x2 x 2 2 x2 x 4 0 x
2
1 21
x2 x 2 3 x
2
1 17 1 21
Y el intervalo es ,
2 2
1 5 1 17 1 21
La solución es 1, ,
2 2 2
11.5
Sumando las tres ecuaciones, y teniendo en cuenta la identidad x x x , tenemos:
x y z x y z x y z 200.0 190.1 178.8
x x y y z z 568.9 2 x 2 y 2 z 568.9 x y z 568.9 / 2 284.45
Ahora, restando la primera ecuación a esta última, obtenemos:
y 0.45
y z 284.45 200.0 84.45
z 84
Así pues, x x x 94 0.65 94.65 , y y y 105 0.45 105.45 ,
z z z 84 0.35 84.35 .
Fuente de la solución: 104 Number Theory Problems From the Training of the USA IMO Team (Andreescu, Andrica, Feng, 2007)
pág. 52.
11.6
n2
Sea an . Está claro que la sucesión a1 , a2 , ... , a2005 es creciente.
2005
Observamos que:
n 12 n2
n 1 n 2 n 1 n n 1 n 2n 1
2
2005 2005 2005 2005 2005
a1 , a2 , ..., a2005 a1 , a2 , ..., a1001 a1002 , a1003 , ..., a2005
2n 1
Si n 1002 2n 2004 2n 1 2005 1
2005
Luego n 1002 an1 an an1 an y por tanto el conjunto a1002 , a1003 , ... , a2005 consta de
2005 1002 1 1004 números diferentes.
2n 1
Si n 1002 2n 2004 2n 1 2005 1, y por tanto los elementos de
2005
a1 , a2 , ..., a1001 recorren todos los valores entre a1 y a1001.
12 10012 1002001
a1 0 , a1001 2005 2005 499.751 499
2005
10022 1004004
a1002 500.75 500 a1001
2005 2005
Luego en a1 , a2 , ... , a1001 hay 500 números diferentes, y en total hay 500 1004 1504
números diferentes.
Fuente de la solución: 104 Number Theory Problems From the Training of the USA IMO Team (Andreescu, Andrica, Feng, 2007) pág. 53.
11.7
Primera versión.
La clave para resolver este problema está en observar la sucesión siguiente:
112 121
1112 12321
11112 1234321
111112 123454321
...
Luego
11111.11 123456789 11111.1111
1
10
0.11 11111.11 123456789 11111.1111 0.1111
1
9
Fuente de la segunda versión: 104 Number Theory Problems From the Training of the USA IMO Team (Andreescu, Andrica, Feng, 2007)
pág. 52.
11.8
2 a2 3 1 2 a 3 2 1 a 2
1 1
1 a 1 a 1 .
2 a
2 a2 3 a2 a2 2
a 1 a 2 a 1 a 2 2 1 a a 2 2 a3 2a 1 0
11.9
4 x 2 51
4 x 2 40x 51 0 4 x 2 51 40x x (*)
40
Observamos que x 1 x x
4 x 2 51
Luego x 1 x
40
10
2 4 6 8 10
4 x 2 51 1.5
Por la parte superior, x 4 x 2 51 40 x x
40 8.5
4 x 2 51 3.5
Por la parte inferior, x 1 x
40 6.5
Así pues, los intervalos en los que puede haber solución son 1.5 x 3.5 y 6.5 x 8.5 .
Vamos a ir comprobando los posibles valores:
4 x 2 51
1.5 x 2 x 1 1 4 x 2 51 40 4 x 2 40 51 imposible
40
4 x 2 51
2 x 3 x 2 2 4 x 2 51 80 4 x 2 80 51 29
40
29
x2 x 2.69 es válido
4
4 x 2 51
3 x 3.5 x 3 3 4 x 2 51 120 4 x 2 69
40
69
x2 x 4.15 no es válido.
4
4 x 2 51
6.5 x 7 x 6 6 4 x 2 51 240 4 x 2 189
40
189
x2 x 6.87 es válido.
4
4 x 2 51
7 x 8 x 7 7 4 x 2 51 280 4 x 2 229
40
229
x2 x 7.56 es válido.
4
4 x 2 51
8 x 8.5 x 8 8 4 x 2 51 320 4 x 2 269
40
269
x2 x 8.20 es válido.
4
29 189 229 269
Las soluciones son x , , ,
2 2 2 2
11.10
x 0 , y 5 (a)
x 5 , y 0 (b)
2 2
x y 25
x 3 , y 4 (c)
x 4 , y 3 (d )
x 0 , y 5 0 x 1 , 5 y 6 (a1 )
a) x 0 , y 5
x 0 , y 5 0 x 1 , 5 y 4 (a2 )
x 5 , y 0 5 x 6 , 0 y 1 (b1 )
b) x 5 , y 0
x 5 , y 0 5 x 4 , 0 y 1 (b1 )
x 3 , y 4 3 x 4 , 4 y 5 (c1 )
x 3 , y 4 3 x 4 , 4 y 3 (c )
c) x 3 , y 4
2
x 3 , y 4 3 x 2 , 4 y 5 (c3 )
x 3 , y 4 3 x 2 , 4 y 3 (c4 )
x 4 , y 3 4 x 5 , 3 y 4 (d1 )
x 4 , y 3 4 x 5 , 3 y 2 (d )
d) x 4 , y 3
2
x 4 , y 3 4 x 3 , 3 y 4 (d 3 )
x 4 , y 3 4 x 3 , 3 y 2 (d 4 )
Es fácil comprobar que esta función satisface las condiciones del enunciado.
Es fácil comprobar que esta función satisface las condiciones del enunciado.
11.12
4 2 16
5 25
2
20 4 20 20 24
4 16
2
20 20 4 20 20 20 24
5 25
2
4 2 16
20 20 20 4 20 20 20 20 24
5 25
2
4 2 16
20 20 20 20 4 (A)
5 25
2
11.13
n 1000 n 1000 n 1000 n 1000 70
70
n
70
n
70
1
70
n 1000 70 n n 1070
Analizando las familias de las gráficas de las funciones anteriores, vemos que la zona solución
será dos intervalos:
n 400
n 1000 70 n n 1000 70 n n 400 , 2500
n 2500
n 508.507
n 1070 70 n 70 n n 1070 n 508 , 2251
n 2251.49
Luego la desigualdad se cumplirá para todos los enteros en 400 , 508 y 2252 , 2500
n 1000
Además se cumple que es entero, es decir, n 1000 es múltiple de 70.
70
En 400 , 508 :
n 400 n 1000 1400 es múltiple de 70
n 1000 1470 n 470
n 1000 1540 n 540 queda fuera del intervalo. Hay dos posibles valores de n.
En 2252 , 2500:
n 2290 n 1000 3290 es múltiple de 70.
n 2290 70 2360 n 1000 3360 es múltiple de 70.
n 2360 70 2430 n 1000 3430 es múltiple de 70.
n 2430 70 2500 n 1000 2600 es múltiple de 70.
11.14
Estudiando la función f ( x) x x vemos que es una función definida a trozos, y en cada
intervalo n, n 1 es una función lineal entre 0 y n. Luego se cortará con la gráfica de a x 2 en
los puntos k xk k 1
Por otro lado, sabemos que kx k x para todo entero k , luego
kx kx kx kx k x k x x kx
Con todo esto podemos suponer que xk k x1 para cierto 1 x1 2 , y por tanto
420 x1 x2 ... x28 x1 2 x1 3x1 ... 28 x1 x1 1 2 3 4 ... 28
420 30
x1 406 x1
406 29
Fuente: [Link]
11.15
x 2 10000 x 10000 x x 2 10000 x 10000 x 10000x
Donde x denota la parte fraccionaria de x . Su gráfica tiene forma de diente de sierra,
tomando valores entre 0 y 10000, y la gráfica de x 2 es la típica parábola. Representando y
comparando estas dos gráficas vemos que se cortarán en 100 valores de x negativos, 98
valores de x positivos y x 0 :
12.1
Primera versión.
5 21 1 106 53
t3
25 20 25 20 250
53 53 53 50
5 1 1
53 50 103
t4 250 50 50
25 21 25 21 25 21 25 21 50 25 21 50
103 103
5 1 1
103 10 103 5 21 50 10
t5 25 21 50 5 21 50 1
25
53 53 5 21 50 53 5 21 50 53
250 10
5353 10 53 101 10 101 101 101
5 21 50 53 5 21 50 53 5 21 5 5 21 5 525
101 101
5 1 1
525 101 21 50 206 21 50 2 21 50
t6 105 1
25
103 103 105 103 105 103 105
25 21 50 21 50
2 21 10
2 100 20
21
5 20 1 5 20 1 101
t6 21
101 101 101
25 25
525 525 21
53 103 101
Luego vemos que hay un bucle de longitud 5: 20 , 21 , , ,
250 25 21 50 525
101
Y puesto que 2020 404 5 , t2020 t5
525
Segunda versión.
Similar a la primera, para ahorrarnos cálculos podemos definir sn 5 tn , y por tanto
5 t 1 sn 1 1 s 1
tn n 1 sn n 1
25 tn 2 5 sn 2 sn 2
Con s1 100 y s2 105 . Ahora hacemos los cálculos:
53 103 101
s3 , s4 , s5 , s6 100 y s7 105 .
50 105 50 105
Y observamos el bucle.
12.2
f (n) f (n 1) f (n 2) n f (n 2) f (n 3) n 1 f (n 2) n
f (n 3) 2n 1
Luego
f (n 3) f (n 6) 2(n 3) 1 f (n 6) 2n 7
Y por tanto:
f (n) f (n 6) 2n 7 2n 1 f (n 6) 2n 7 2n 1 f (n 6) 6
Así pues, por cada vez que puedo restar 6, sumo 6, y puesto que 2018 336 6 2
Puedo restar 336 veces, y por tanto
f (2018) f (2) 336 6 1 2016 2017 (B)
12.4
Paso 1 OA: 0
y 3 5
Paso 2 AB: sin 60º y 3
5/ 2 2 4
y 3 5
Paso 3 BC: cos 30º y 3
5/ 4 2 8
Paso 4 CD: 0
y 3 5
Paso 5 DE: sin 60º y 3
5 / 16 2 32
y 3 5
Paso 6 EF: cos 30º y 3
5 / 32 2 64
5 21 5 21 5 21 5 21 5 21 1 1 1
3 8 3 14 3 20 3 ... 2 3 1 6 12 18 ...
2 2
16 2 16 2 16 2 16 2 16 2 2 2
21 5 n
5 1 1 1 21 1 5 21 64 5
2 3 1 6 ... 2 3 6 2 3
2
16 2 2 6
2
2 6 3
2 16 n0 2 2 16 63 3
Veamos ahora su comportamiento en horizontal.
Paso 1 OA: 5
x 1 5
Paso 2 AB: cos 60º x
5/ 2 2 4
x 1 5
Paso 3 BC: sin 30º x
5/ 4 2 8
5
Paso 4 CD: x
8
x 1 5
Paso 5 DE: cos 60º x
5 / 16 2 32
x 1 5
Paso 6 EF: sin 30º x
5 / 32 2 64
5 5 5 5 5 315
Luego se ha producido un cambio total de 5
4 8 8 32 64 64
1 315
En el siguiente grupo de 6 pasos será
26 64
n
1 315 315 64
Y el resultado final será 6 5
n 0 2 64 64 63
5
Así pues, el gusano se acerca más y más al punto 5 ,
3
12.5
Vamos viendo como se comporta la sucesión:
a2 4 9 1 196
2
a3 1 9 6 1 17 2 289
2
a4 2 8 9 1 202 400
2
a5 4 0 0 1 52 25
2
a6 2 5 1 82 64
2
a7 6 4 1 112 121
2
a8 1 2 1 1 52 25 a5
2
12.6
6
n2
3n
n 2
3n
2
3n
1
3n
1
n / 2
1 1
1 e 6 1
n n n n (n / 2) (n / 2) e6
La respuesta correcta es la (C)
12.7
an a0 r n . Vamos a suponer que la sucesión es creciente, es decir, que r 1 .
12 an an1 a0 r n a0 r n 1 a0 r n 1 r 1
12
a0 r n1
r 1
3 an an1 a0 r n a0 r n1 a0 r n1 r 1
3
a0 r n 1
r 1
Luego
12 3
12(r 1) 3(r 1) 12r 12 3r 3 12r 3r 3 12
r 1 r 1
15 5
9r 15 r
9 3
12.8
1
a log 8 (2 x) log 8 (2) log 8 ( x) log 23 (2) log 8 ( x) log 8 ( x)
3
log 8 x log8 x log 8 x 3
b log 4 x log 8 x
log 8 4 log 23 22 2/3 2
log 8 x log 8 x log 8 x
c log 2 x 3 log 8 x
log 8 2 log 23 2 1/ 3
12.9
Teniendo en cuenta que 289 172 , hacemos el cambio de variable u 217 , y por tanto
2289 1 u17 1
217 1 u 1
Ahora observamos que tenemos una suma de una sucesión geométrica de razón 2:
u 1 217 1 216 215 ... 2 1
Luego, finalmente:
2289 1
2 1
17
216 215 ... 21 20 21715 21713 21711 ... 2173 2171 20
12.10
En principio podríamos creer que la respuesta correcta es 9, puesto que
1 2 3 ... 9 45
Pero debemos observar que podemos trabajar también con números negativos, y ver que
89 88 87 ... 89 90 90
Luego podemos encontrar una suma de 90 términos.
Aunque no es apropiado en un contexto de una competición matemática, se puede demostrar
que 90 es la solución máxima:
45 x x 1 x 2 ... x (n 1) n x 1 2 3 ... (n 2) (n 1)
n(n 1)
nx 90 2n x n(n 1) n2 x n 1 n | 90 90 90
2
12.11
Primera versión.
Calculamos a mano los primeros valores de esta sucesión:
3 3 1 3 3 1 3 3
a1 1 , a2 , a3 , a 4 , a5 , a6 , a 7 , a8 , a9
7 11 5 19 23 9 31 35
Y observamos que siguen una pauta en grupos de tres en tres: Sea N (n 1) / 3 ,
1 3 3
, ,
4 N 1 12 N 7 12 N 11
2019 1 672 3 2 , luego nos piden el tercer elemento del grupo N 672 , es decir:
3 3 3
a 2019
12 N 11 12 672 11 8075
12.12
0.23k 0.232323...k 2 k 1 3 k 2 2 k 3 3 k 4 2 k 5 3 k 6 ...
n 1
2 n k 2
1
2k 3 k 1 2k 3 2 1 2k 3 2
n 0 1 k 1 k
Observación. Nos podemos “saltar” esta última ecuación, demasiado larga sin calculadora,
aprovechando que la pregunta es de tipo multirespuesta, con lo que todo se reduce a determinar
qué opción de entre las cinco propuestas satisface dicha ecuación.
12.13
Fijando un i tenemos:
100
100 101
(i j ) i 1 i 2 i 3 ... i 100 100i 1 2 3 ... 100 100i
j 1 2
100 101
100i 100i 5050
2
12.14
Queremos calcular la siguiente serie: 2 3 5 a b c
0 a ,b , c
Si fijamos un b y un c , tenemos:
1 1 1 1 1 1 1
a b c
0 a ,b , c 2 3 5
a b c a b c b c
0 a 2 3 5 0 a 2 3 5 35
0 a 2
a
b c
3 5 1 (1 / 2)
1
2 2 b1 c
b c
35 35
Y la solución es 15 4 19
13.1
5x! 3!4! 3!4 3! 3!4 1 3!5 x! 3! 3! x 3
5
5
13.2
7! 6! 5! 5!7 6 6 1 7 6 6 1 49 7
7! 6! 5! 5!7 6 6 1 7 6 6 1 35 5
13.3
1 1 10 9 10 90 10 90 10 100 x
x 100
8! 9! 10 9 8! 10 9! 10! 10! 10! 10! 10!
13.4
2 3 4 5 N! 5! N! N!
5! 9! 12 N! 3 4 N! 9! 10! 10 9! N! N 10 (A)
5 2 10 10
Recopilatorio de identidades notables.
1. Diferencia de cuadrados: a 2 b2 (a b)(a b)
2. Cuadrado de sumas y restas: (a b)2 a 2 2ab b2
(a b)2 a 2 2ab b2
3. Cubo de sumas y restas: a b3 a3 3ab(a b) b3
a b3 a3 3ab(a b) b3
4. Cuadrado de la suma de 3 números: a b c a 2 b2 c 2 2ab bc ca
2
c) 13 23 33 ... n3 n (n 1) / 2
2
c) 1 3 5 ... (2n 1) n2
d) 12 32 52 ... (2n 1) 2 n (2n 1) (2n 1) / 3
e) 1 2 2 3 3 4 ... n(n 1) n(n 1)(n 2) / 3
r n 1 1
f) 1 r r 2 r 3 ... r n (Suma de una progresión geométrica)
r 1
a
La serie geométrica: a r n (si r 1)
k 0 1 r